Download as pdf or txt
Download as pdf or txt
You are on page 1of 149

6/9/24, 12:55 PM Neet PG Preparation, Neet PG Coaching, FMGE, USMLE

Grand Test-28 (Clinical New Pattern)

RESULT

View View View SectionWise


Result
leaderboard Analysis Solution Analysis

Grand Test-28 (Clinical New Pattern)


Question Wise Report

Question: 1
Siewert classification is used for?

A Esophageal cancer

B Stomach cancer

C GE junction cancer

D Periampullary cancer

39.26% People got this right

Explanation:

Correct Answer (C)

https://emedicoz.com/testresult/683510s14692521 1/149
6/9/24, 12:55 PM Neet PG Preparation, Neet PG Coaching, FMGE, USMLE

Question: 2
Which of the following is not a capsular plate of liver?

A Portal plate

B Hilar plate

C Cystic plate

D Umbilical plate

9.42% People got this right

Explanation:

Correct Answer (A)

https://emedicoz.com/testresult/683510s14692521 2/149
6/9/24, 12:55 PM Neet PG Preparation, Neet PG Coaching, FMGE, USMLE

Glisson’s capsule which covers the liver extends into the liver at the hilus and
covers the portal triad, where it is called Glisson’s sheath. Glisson’s capsule also
covers the Glissonian pedicles inside the liver. Couinaud called this sheath the
Valoean sheath, after Valoeus, an anatomist from the Middle Ages who first
described the liver capsule.

The plate system consists of bile ducts and blood vessels surrounded by a
sheath. There are three plates in the hilar area: the hilar plate, the cystic plate,
and the umbilical plate. The bile duct and blood vessel branches penetrate the
plate system and form Glisson's capsule in all segments of the liver, except for
the medial segment.

The hilar plate is the most important plate in liver surgery.

Question: 3
All of the following are correct regarding Emphysematous cholecystitis
except?

A More common in Males

B D M is a risk factor

C Usually associated with


acalculus cholecystitis

D Most common causative


organism is Pseudomonas

28.93% People got this right

Explanation:

Correct Answer (D) Most common organism is clostridia not pseudomonas.

https://emedicoz.com/testresult/683510s14692521 3/149
6/9/24, 12:55 PM Neet PG Preparation, Neet PG Coaching, FMGE, USMLE

Question: 4
Which of the following structure causes sharp pain while doing
abdominal surgery?

A Parietal peritoneum

B Liver parenchyma

C Small intestine

D Colon

59.68% People got this right

Explanation:

Correct Answer (A) Parietal peritoneum causes sharp shooting somatic pain all
other structure cause visceral pain

Question: 5
A 32 year old patient presented with Jaundice, pruritis and clay coloured
stool. USG showed Gall stone and dilated CBD. Which of the following
finding is consistent with this condition?

A High unconjugated
bilirubin

B Very High SGOT

C Increased GGT

D Positive urobilinogen in

https://emedicoz.com/testresult/683510s14692521 4/149
6/9/24, 12:55 PM Neet PG Preparation, Neet PG Coaching, FMGE, USMLE

urine

34.94% People got this right

Explanation:

Correct Answer (C) Clinical feature is suggestive of obstructive jaundice which


should have high conjugated bilirubin, high ALP and high GGT. While high
unconjugated bilirubin, SGOT and SGPT is suggestive of hepatocellular jaundice.

Question: 6
A 70 year old male was admitted with large bowel obstruction and caecal
diameter of 12.5 cm. An urgent laparotomy was done and a completely
obstructing rectal cancer was found in upper 1/3rd of rectum. Patient is
hemodynamically stable with a known H/O of DM and HT which is well
controlled on medicatiion. What is the best treatment for this patient?

A Defunctioning colostomy

B Resection and anastomosis

C Resection and Hartmann’s


procedure

D Abdomino-perineal
resection

42.86% People got this right

Explanation:

Correct Answer (C) Resection and Hartmann’s procedure

Question: 7
Spigelian hernia occurs at?
https://emedicoz.com/testresult/683510s14692521 5/149
6/9/24, 12:55 PM Neet PG Preparation, Neet PG Coaching, FMGE, USMLE

A Below umbilicus behind


rectus muscle

B Above umbilicus behind


rectus muscle

C Lateral border of rectus


abdominis

D Through lacunar ligament

37.67% People got this right

Explanation:

Correct Answer (C)

Question: 8
False statement about RIRS is?

A Used for < 2cm stones

B Can be used for lower


calyceal stones

https://emedicoz.com/testresult/683510s14692521 6/149
6/9/24, 12:55 PM Neet PG Preparation, Neet PG Coaching, FMGE, USMLE

C Done by flexible
ureteroscopy

D Stone is broken with the


help of KTP laser

18.84% People got this right

Explanation:

Correct Answer (D) Ho: YAG laser is used for stone breaking not KTP laser RIRS
(retrograde intrarenal surgery): is a procedure for upper tract stones using
flexible fibreoptic endoscope. It is done in retrograde fashion i.e. through the
urethra into the bladder and then through the ureter into the urine-collecting
part of the kidney. It is approved for stones which are < 2cm in size and is
considered a good minimal invasive technique for pelvic and calyceal stones.

Question: 9
Surgery shown in the picture is done for?

A Pyelolithotomy

B PUJ obstruction

C TCC of upper ureter/ Pelvis

https://emedicoz.com/testresult/683510s14692521 7/149
6/9/24, 12:55 PM Neet PG Preparation, Neet PG Coaching, FMGE, USMLE

D Extended pyelolithotomy

51.37% People got this right

Explanation:

Correct Answer (B) Anderson Hynes pyeloplasty is a type of dismembered


pyeloplasty done for PUJ obstruction. Procedures for PUJ obstruction are: •
Anderson-Hynes dismembered pyeloplasty (Most commonly done surgery with
best results). • Antegrade or retrograde endopyelotomy. • The Foley Y-V plasty is
useful for the high insertion variant. • Spiral and vertical flaps, such as Culp and
DeWeerd and Scardino and Prince, are useful when a long-strictured segment of
diseased ureter is encountered. • Ureterocalicostomy, anastomosis of the ureter
to a lower pole renal calyx, is most often reserved for failed open pyeloplasty
where there is no extrarenal pelvis.

Question: 10
Which of the following is not a surgery for Hypospadias

A Dennis Brown operation

B MAGPAI

C Matheus

D Foley’s Y V plasty

20.81% People got this right

Explanation:

Correct Answer (D) Foley Y V plasty is done for PUV obstruction.

Question: 11

https://emedicoz.com/testresult/683510s14692521 8/149
6/9/24, 12:55 PM Neet PG Preparation, Neet PG Coaching, FMGE, USMLE

True statement about the surgery shown in the image is?

A LABG - purely restrictive

B LABG – restrictive and


malabsorptive

C VBG - purely restrictive

D VBG – Restrictive and


malabsorptive

33.64% People got this right

Explanation:

Correct Answer (A) LABG - purely restrictive

Question: 12
Most sensitive investigation for DCIS is?

A MRI

B Mammography

https://emedicoz.com/testresult/683510s14692521 9/149
6/9/24, 12:55 PM Neet PG Preparation, Neet PG Coaching, FMGE, USMLE

C USG

D CT scan

41.18% People got this right

Explanation:

Correct Answer (A) MRI

Question: 13
Not a feature of Lobular carcinoma?

A Usually Multicentric and


multifocal

B LCIS is more easily


detected on Mammography then
DCIS

C CDH 1 gene

D Signet cell may be found

30.75% People got this right

Explanation:

Correct Answer (B) LCIS is more easily detected on Mammography then DCIS

Question: 14
Preferred treatment for medullary cancer thyroid is?

https://emedicoz.com/testresult/683510s14692521 10/149
6/9/24, 12:55 PM Neet PG Preparation, Neet PG Coaching, FMGE, USMLE

A Surgery and radiotherapy

B Surgery and
chemotherapy

C Radio and Chemotherapy

D Only surgery

13.94% People got this right

Explanation:

Correct Answer (D) Total thyroidectomy and central node dissection is the
standard treatment for medullary cancer.

Question: 15
False statement about secondary hyper-parathyroidism is?

A Associated with CRF

B Associated
Hyperphosphatemia

C Hypercalcemia is a
consistent feature

D Can cause calciphylaxis

31.62% People got this right

Explanation:

https://emedicoz.com/testresult/683510s14692521 11/149
6/9/24, 12:55 PM Neet PG Preparation, Neet PG Coaching, FMGE, USMLE

Correct Answer (C) Secondary hyper-parathyroidism is associated with CRF and


induced by Hyperphosphatemia and hypocalcemia.

Question: 16
Malignant pheochromocytoma maximally produces?

A Dopamine

B Epinephrine

C Nor epinephrine

D Metanephrines

11.44% People got this right

Explanation:

Correct Answer (A) Dopamine

Question: 17
Varicose vein. Test for SFJ incompetence?

A Trendelenburg test

B Fegan test

C Tourniquet test

D Perthes test

https://emedicoz.com/testresult/683510s14692521 12/149
6/9/24, 12:55 PM Neet PG Preparation, Neet PG Coaching, FMGE, USMLE

48.01% People got this right

Explanation:

Correct Answer (A)

Question: 18
Nicoladoni Branham sign is seen in?

A AV fistula

B DVT

C Acute mesenteric ischemia

D Popliteal aneurysm

30.47% People got this right

Explanation:

Correct Answer (A) Nicoladoni Branham sign is seen in AV fistula. When the
artery proximal to the fistula is compressed, blood is no longer shunted from
artery to vein side which leads to increased peripheral vascular resistance. This
causes relative hypertension and bradycardia. Locally, because no blood is being
shunted, the swelling reduces and bruit/thrill disappears.

https://emedicoz.com/testresult/683510s14692521 13/149
6/9/24, 12:55 PM Neet PG Preparation, Neet PG Coaching, FMGE, USMLE

Question: 19
What is the treatment of open pneumothorax?

A IPPV

B ICD with underwater seal

C Tube thoracostomy and


close the rent

D Air-tight dressing

28.45% People got this right

Explanation:

Correct Answer (C)

An open pneumothorax happens when air builds up in the pleural cavity, due to
a hole in the chest wall. It differs from a closed pneumothorax, during which air
enters the pleural cavity from the lungs themselves. An open pneumothorax can
occur as a result of direct trauma or could be iatrogenic.
An open pneumothorax is treated by applying a three-way dressing to the
wound, which involves taping the dressing on only three of the edges. The intent
is to prevent the individual from breathing in air through the opening in the
chest wall while allowing them to exhale air from their lungs. An inserted chest
tube will further assist the individual in breathing. If the dressing is not properly

https://emedicoz.com/testresult/683510s14692521 14/149
6/9/24, 12:55 PM Neet PG Preparation, Neet PG Coaching, FMGE, USMLE

applied, and it allows air into the chest space without allowing it to escape, a
tension pneumothorax may develop.

Question: 20
False stamen about superficial 2nd degree burn?

A Painful

B Heals in 4 weeks time.

C Blisters found

D Heals by epithelization
from skin appendages

27.01% People got this right

Explanation:

Correct Answer (B) Superficial 2nd degree burn heals in 2 weeks not 4 weeks.

Question: 21
Which cancer does not spread through neural sheath?

A Adenoid cystic tumour of


salivary gland.

B Carcinoma Gall bladder

C Adenocarcinoma of
pancreas

https://emedicoz.com/testresult/683510s14692521 15/149
6/9/24, 12:55 PM Neet PG Preparation, Neet PG Coaching, FMGE, USMLE

D Hepatocellular cancer

20.62% People got this right

Explanation:

Correct Answer (D) Following tomours have property of perineural spread-


Adenoid cystic tumour of salivary gland, Carcinoma Gall bladder,
Adenocarcinoma of pancreas, Cholangiocarcinoma.

Question: 22
In post radical neck dissection shoulder syndrome all are seen except?

A Pain

B Restricted range of
movement

C Drooping of shoulder

D Weakness of deltoid
muscle.

29.07% People got this right

Explanation:

Correct Answer (D) I radical neck dissection Spinal accessory nerve is sacrifice
which leads to denervation of trapezius called shoulder syndrome (not deltoid).
It leads to inability of abduction beyond 90 degree along with internal rotation,
long standing pain in shoulder and drooping of shoulder

Question: 23

https://emedicoz.com/testresult/683510s14692521 16/149
6/9/24, 12:55 PM Neet PG Preparation, Neet PG Coaching, FMGE, USMLE

In which of the following disease the overall survival rate is increased by


screening procedure?

A Lung cancer

B Colon cancer

C Prostate cancer

D Breast cancer

16.15% People got this right

Explanation:

Correct Answer (B) Colon cancer

Question: 24
Most common nosocomial infection is?

A Surgical site infection

B Respiratory tract infection

C UTI

D Skin and soft tissue


infection

26.72% People got this right

Explanation:

Correct Answer (C)

https://emedicoz.com/testresult/683510s14692521 17/149
6/9/24, 12:55 PM Neet PG Preparation, Neet PG Coaching, FMGE, USMLE

UTI

Question: 25
Identify the instrument?

A Jolls thyroid retractor

B Gelpi Retractor

C Weitlaner retractor

D Gutow retractor

51.23% People got this right

Explanation:

Correct Answer (A)

Jolls thyroid retractor

https://emedicoz.com/testresult/683510s14692521 18/149
6/9/24, 12:55 PM Neet PG Preparation, Neet PG Coaching, FMGE, USMLE

Question: 26
A primipara wishes to know the symptoms of onset of labour. Which of
the following is the most correct answer to her question?

A Blood-stained mucous
vaginal discharge.

B Colicky lower abdominal


pain and backache which increase
in frequency, intensity and
duration with time.

C Colicky lower abdominal


pain and backache which is not
relieved by analgesics.

D Dribbling

54.2% People got this right

Explanation:

Correct Answer (B) Colicky lower abdominal pain and backache which increase in
frequency, intensity and duration with time. The definitive symptom of labour is
colicky lower abdominal pain and backache which increase progressively. This
should be accompanied by progressive effacement and dilatation of the cervix.

https://emedicoz.com/testresult/683510s14692521 19/149
6/9/24, 12:55 PM Neet PG Preparation, Neet PG Coaching, FMGE, USMLE

The pain is not relieved by common analgesics. Show or dribbling may or may
not be present at the onset of labour.

Question: 27
Which one of the following is not a sign of separation of placenta?

A Fundus of the uterus rises


to the umbilicus

B The uterine is well


contracted and retracted

C The cord "lengthens", but


recedes on pushing the uterus
upwards

D Sudden gush of bleeding

34.26% People got this right

Explanation:

Correct Answer (C) The cord "lengthens", but recedes on pushing the uterus
upwards signs: 1-umbilical cord lengthens and it does not recede back when
uterus is pushed upwards. -The most reliable sign is the lengthening of the
umbilical cord as the placenta separates and is pushed into the lower uterine
segment by progressive uterine retraction. 2-gush of blood 3-firm/globular
uterus 4-uterus rises to the abdominal wall 5- supra pubic bulge

Question: 28
In deep transverse arrest, the head is arrested at the levels of:

A Ischial tuberosity

https://emedicoz.com/testresult/683510s14692521 20/149
6/9/24, 12:55 PM Neet PG Preparation, Neet PG Coaching, FMGE, USMLE

B Ischial spine

C Inlet of pelvis

D Perineum

56.7% People got this right

Explanation:

Correct Answer (B) Ischial spine it is the narrowest plane in the pelvis and the
baby is not able to pass through this station.

Question: 29
Which one of the following is the most practiced regime of antenatal
corticosteroid therapy for maturation of fetal:

A Dexamethasone 12 mg
daily for 3 days

B Betamethasone 12 mg 12
hourly × 2 dose

C Dexamethasone 6 mg 12
hourly × 4 dose

D Betamethasone 8 mg 8
hourly × 3 dose

63.05% People got this right

Explanation:

https://emedicoz.com/testresult/683510s14692521 21/149
6/9/24, 12:55 PM Neet PG Preparation, Neet PG Coaching, FMGE, USMLE

Correct Answer (C) Dexamethasone 6 mg 12 hourly × 4 dose Total dose of 24mg


is to be covered in 48 hours. It is to be given in a preterm labour of 24 to 36+6
weeks.

Question: 30
A 19-year-old comes to an adolescent clinic with complain of nausea and
vomiting. She did a home urine pregnancy test which was positive. She
does not remember her date of last menstrual period. US shows a viable
pregnancy of 8 weeks gestation. Which of the following statements
regarding first trimester ultrasound is correct?

A A gestational sac can be


first seen 2 weeks after LMP

B The accuracy of
determining gestational age using
ultrasound begins to decrease
after first trimester

C Yolk sac is the first sign of


pregnancy on USG

D True gestational sac is


always centrally located in the
uterine cavity

43.63% People got this right

Explanation:

Correct Answer (B) The accuracy of determining gestational age using


ultrasound begins to decrease after first trimester the accuracy is highest if the
scan is a first trimester scan and the parameter used is CRL.

Question: 31

https://emedicoz.com/testresult/683510s14692521 22/149
6/9/24, 12:55 PM Neet PG Preparation, Neet PG Coaching, FMGE, USMLE

A third para, who had a normal vaginal delivery in her first pregnancy and
an upper segment caesarean section for transverse lie in her second
pregnancy, attends the antenatal clinic at 36 weeks. The presentation is
cephalic. The estimated fetal weight is 2.8 kg. The birth weight of the first
baby was 3.2 kg. There are no other pregnancy complications. What is the
most appropriate method of management?

A Await spontaneous onset


of labour till 40 weeks.

B Induce labour with


amniotomy followed by oxytocin
infusion.

C Perform an elective lower


segment caesarean section at 37
weeks.

D Perform an elective lower


segment caesarean section at 39
weeks.

31.57% People got this right

Explanation:

Correct Answer (C) Perform an elective lower segment caesarean section at 37


weeks. Having an upper segment caesarean section in past would be a
contraindication for VBAC delivery. We must do elective LSCS and that too at 37
weeks because there is a risk of silent rupture in the third trimester in such cases.

Question: 32
An ultrasound scan is performed in a woman with a monochorionic
diamniotic twin pregnancy at 24 weeks. Bladder of one twin not visible
with oligohydramnios and there is polyhydramnios in the other sac.
Doppler studies are normal. What is the best management option?

https://emedicoz.com/testresult/683510s14692521 23/149
6/9/24, 12:55 PM Neet PG Preparation, Neet PG Coaching, FMGE, USMLE

A Perform amnioreduction of
the sac with polyhydramnios.

B Perform Doppler studies


weekly.

C Perform fetoscopic laser


ablation of communicating blood
vessels.

D Perform selective feticide


of the twin with oligohydramnios.

46.76% People got this right

Explanation:

Correct Answer (C) Perform fetoscopic laser ablation of communicating blood


vessels. This is a case of TWIN-TWIN TRANSFUSION SYNDROME. The stage is
stage 2 of Quintero classification. The treatment of choice is laser ablation of AV
malformation in the placenta.

Question: 33
Indications for caesarean section in pregnancy are all except:

A Eisenmenger syndrome

B Aortic coarctation

C Aortic aneurysm

D Aortic dissection

22.3% People got this right

https://emedicoz.com/testresult/683510s14692521 24/149
6/9/24, 12:55 PM Neet PG Preparation, Neet PG Coaching, FMGE, USMLE

Explanation:

Correct Answer (A) Eisenmenger syndrome Aortic lesions like dissection,


aneurysms, coarctations and stenosis are contraindications for vaginal delivery.
Eisenmenger syndrome is an indication of MTP but if the patient denies MTP
and wishes to continue pregnancy then she is allowed to continue pregnancy.
The risk is same with vaginal delivery and caesarean section in this case so it is
not an indication of caesarean section.

Question: 34
In heart patient the worst prognosis during pregnancy is seen in:

A Mitral regurgitation

B Mitral valve prolapse

C Aortic stenosis

D Pulmonary stenosis

32.48% People got this right

Explanation:

Correct Answer (C) Aortic stenosis Stenotic lesions are worse than regurgitant
lesions. Aortic stenosis has highest mortality amongst all the given options.
Eisenmenger syndrome has even higher mortality but that is not given in the
option.

Question: 35
Infants of diabetic mothers are likely to have the following cardiac
anomaly:

A Coarctation of aorta

https://emedicoz.com/testresult/683510s14692521 25/149
6/9/24, 12:55 PM Neet PG Preparation, Neet PG Coaching, FMGE, USMLE

B Fallot's tetralogy

C Ebstein's anomaly

D Transposition of great
arteries

47.67% People got this right

Explanation:

Correct Answer (D) Transposition of great arteries CVS anomalies are the most
common anomalies in the infants of the diabetic mothers. The most common
ones are VSD, TGA, endocardial cushion defects, etc.

Question: 36
A primigravida 32-year-old female is admitted to the antenatal ward at 35
weeks is found to BP of 160/110. Her urine does not contain albumin.
Which of the following drugs should not be used in her management?

A Furosemide

B Labetalol

C Nifedipine

D Hydralazine

54.78% People got this right

Explanation:

https://emedicoz.com/testresult/683510s14692521 26/149
6/9/24, 12:55 PM Neet PG Preparation, Neet PG Coaching, FMGE, USMLE

Correct Answer (A) Furosemide The drug of choice is labetalol. Furosemide must
not be given because it causes intra vascular dehydration and in this case, there
is already a reduced level of intra vascular volume.

Question: 37
All of the following increase in pre-eclampsia except:

A Uric acid

B Thromboxane A2

C sFIt-1

D VEGF

26.29% People got this right

Explanation:

Correct Answer (D) VEGF Uric acid is raised due to reduced functions of kidney.
TXA2 and sFlt-1 are vasoconstrictors. VEGF is vasodilator and it is having
reduced activity in pre-eclampsia.

Question: 38
A 45-year-old woman presented with lower abdominal discomfort on
examination uterus appears uniformly enlarged and adnexa is free.
Histopathology of uterus shows endometrial glands inside the
myometrium. The diagnosis is:

A Adenomyosis

B Endometriosis

https://emedicoz.com/testresult/683510s14692521 27/149
6/9/24, 12:55 PM Neet PG Preparation, Neet PG Coaching, FMGE, USMLE

C Leiomyoma

D Endometrial Hyperplasia

58.96% People got this right

Explanation:

Correct Answer (A) Adenomyosis Adenomyosis means ectopic, functional


endometrium inside the myometrial tissue. It is also called as endometriosis
interna.

Question: 39
Ideal contraceptive for lactating mother is:

A Barrier method

B Combined OCP

C Lactational amenorrhoea

D Progesterone only pill

40.03% People got this right

Explanation:

Correct Answer (D) Progesterone only pill Barrier has high failure rate, COCs will
reduce the milk output, LAM is a natural method with 5 % failure rate. POP will
be best for lactating mothers.

Question: 40
Peritoneum is opened in all of the following procedures except:

https://emedicoz.com/testresult/683510s14692521 28/149
6/9/24, 12:55 PM Neet PG Preparation, Neet PG Coaching, FMGE, USMLE

A Mini laparotomy

B Laparoscopy

C Vasectomy

D Transvaginal tubectomy

35.66% People got this right

Explanation:

Correct Answer (C) Vasectomy Vasectomy does not require opening of the
peritoneum. Other procedures require this step.

Question: 41
Contraceptive to be avoided in epilepsy:

A OCPs

B Condom

C IUCD

D Mirena

53.77% People got this right

Explanation:

Correct Answer (A) OCPs COC pills are not given if patient is on anti-epileptic
medications because estrogen is an enzyme inducer and it reduces the amount
of anti-epileptic drug in the circulation.

https://emedicoz.com/testresult/683510s14692521 29/149
6/9/24, 12:55 PM Neet PG Preparation, Neet PG Coaching, FMGE, USMLE

Question: 42
A mother brings her 19 years old daughter to your clinic with complaint
that she has not started having menses. General examination reveals
normally developed breasts and pubic hair. On pelvic examination,
vaginal ending is blind and uterus is not palpable. Which of the following
do you suspect?

A Mullerian agenesis

B Ashermann syndrome

C Gonadal dysgenesis

D Turner syndrome

68.33% People got this right

Explanation:

Correct Answer (A) Mullerian agenesis Uterus is present in ASHERMANN


syndrome. Secondary sexual characters are absent in gonadal dysgenesis. Turner
syndrome is a case of gonadal dysgenesis. In a case of Mullerian agenesis, uterus
is absent and ovaries are present and so the secondary sexual characters are
present as well.

Question: 43
Which of the following is the most common route of development of
genital tuberculosis?

A Hematogenous

B Lymphatics

https://emedicoz.com/testresult/683510s14692521 30/149
6/9/24, 12:55 PM Neet PG Preparation, Neet PG Coaching, FMGE, USMLE

C Ascending infection

D Direct mode

52.93% People got this right

Explanation:

Correct Answer (A) Hematogenous Lungs are the most common site from where
TB is acquired by the genital tract. Fallopian tube is the most common organ
involved.

Question: 44
True regarding testicular feminization syndrome:

A Phenotype that of female,


genotype is 46XX

B Phenotype and genotype


of male

C Genotype 46XY, phenotype


is of female and gonads of male

D Gonads and genotype that


of female

69.58% People got this right

Explanation:

Correct Answer (C) Genotype 46XY, phenotype is of female and gonads of male
it is a case of androgen insensitivity syndrome. It is the most common cause of
male intersex aka male pseudohermaphrodite. In this case, the genotype is male

https://emedicoz.com/testresult/683510s14692521 31/149
6/9/24, 12:55 PM Neet PG Preparation, Neet PG Coaching, FMGE, USMLE

but the phenotype is female because there are no androgenic receptors. Testes
are present in the pelvis or in the inguinal canal.

Question: 45
Which of these is diagnostic of menopause?

A Serum FSH > 40 IU/L

B Serum LH > 20 IU/L

C Serum FSH < 40 IU/L

D Serum oestradiol < 300pg


per ml

16.36% People got this right

Explanation:

Correct Answer (A) Serum FSH > 40 IU/L Menopause is a case of ovarian failure.
In this case, the follicles are depleted and so the estrogen and progesterone
levels are reduced. Due to this, there is an increase in the levels of FSH and LH. It
is a case of hypergonadotropic hypogonadism. Biochemical definition of
menopause will be Sr. FSH >=40 IU/L.

Question: 46
HRT is helpful in all of the following except:

A Vaginal atrophy

B Flushing

https://emedicoz.com/testresult/683510s14692521 32/149
6/9/24, 12:55 PM Neet PG Preparation, Neet PG Coaching, FMGE, USMLE

C Osteoporosis

D Coronary heart disease

59.21% People got this right

Explanation:

Correct Answer (D) Coronary heart disease HRT is not useful in preventing
coronary heart disease. It is responsible for an increase in the risk of
thromboembolic phenomenon.

Question: 47
A young lady presents to your office with complain of copious vaginal
discharge, but there is no cervical discharge on per speculum
examination. Which of the following should be given for the
management?

A Metronidazole and
fluconazole

B Metronidazole and
azithromycin

C Metronidazole and
doxycycline

D Fluconazole only

38.73% People got this right

Explanation:

Correct Answer (A) Metronidazole and fluconazole The most common vaginal
infections are anaerobic, fungal and protozoal. The combination therapy of

https://emedicoz.com/testresult/683510s14692521 33/149
6/9/24, 12:55 PM Neet PG Preparation, Neet PG Coaching, FMGE, USMLE

metronidazole with fluconazole or secnidazole with fluconazole will be


appropriate.

Question: 48
A 55-year-old woman was found to have Ca cervix, FIGO stage 2B. What
would be the management?

A Surgery plus chemotherapy

B Radiotherapy plus
chemotherapy

C Chemotherapy

D Radiotherapy plus HPV


vaccine

28.45% People got this right

Explanation:

Correct Answer (B) Radiotherapy plus chemotherapy Surgery is not done in an


advanced cervical cancer. The mode of treatment in an advanced cancer of
cervix is chemoradiation. Stage 2B, 3 and 4 as well as bulky early cancers are
treated with chemoradiation therapy.

Question: 49
Nulliparous women have high risk of following cancer:

A Cervical cancer

B Vaginal cancer

https://emedicoz.com/testresult/683510s14692521 34/149
6/9/24, 12:55 PM Neet PG Preparation, Neet PG Coaching, FMGE, USMLE

C Ovarian cancer

D Endometrial Ca

49.33% People got this right

Explanation:

Correct Answer (D) Endometrial Ca The best answer is endometrial cancer. Less
progesteronic exposure is surely a high risk of endometrial cancer. Multiparity is
protective for endometrial cancer.

Question: 50
The reference part in POP -Q classification is:

A Hymen

B Internal os

C Ischial spine

D External os

45.92% People got this right

Explanation:

Correct Answer (A)

Hymen

https://emedicoz.com/testresult/683510s14692521 35/149
6/9/24, 12:55 PM Neet PG Preparation, Neet PG Coaching, FMGE, USMLE

Question: 51
Which of the following is not true regarding ferroptosis?

A Cell membrane damage

B Mitochondrial shrinkage

C Pyknosis

D Increased free radicals

17.95% People got this right

Explanation:

Correct Answer (C) Pyknosis. Nuclear morphology remains normal in ferroptosis.

Question: 52
Which of the following collagen types is seen predominantly in keloid?

A Type I

https://emedicoz.com/testresult/683510s14692521 36/149
6/9/24, 12:55 PM Neet PG Preparation, Neet PG Coaching, FMGE, USMLE

B Type IV

C Type III

D Type II

5.57% People got this right

Explanation:

Correct Answer (A) Type I. In keloid there is have haphazardly arranged normal
collagen [type 1>type 3]. In hypertrophic scar there is delayed maturation of
collagen from type 3 to type 1 thus type 3>1 in HS.

Question: 53
Which of the following is not feature of severe combined
immunodeficiency?

A Absent thymic shadow

B Shrunken Spleen

C Absent Immunoglobulins

D Hypoclcemia

42.61% People got this right

Explanation:

Correct Answer (D) Hypocalcemia. Hypocalcemia is a manifestation of Di George


syndrome. In SCID there is absent T and B cells thus absent Ig, atrophied thymus
spleen, LN.

https://emedicoz.com/testresult/683510s14692521 37/149
6/9/24, 12:55 PM Neet PG Preparation, Neet PG Coaching, FMGE, USMLE

Question: 54
What is likely diagnosis in this patient?

A Amyloidosis

B Right heart failure

C Splenic vein thrombosis

D Malignant hypertension

36.8% People got this right

Explanation:

Correct Answer (B) The image shows Nutmeg liver caused due to congestion in
liver. Right heart failure is a cause of the same.

Question: 55
Which of the following disease is due to paternal disomy of chromosome
15?

A Angelmann syndrome

https://emedicoz.com/testresult/683510s14692521 38/149
6/9/24, 12:55 PM Neet PG Preparation, Neet PG Coaching, FMGE, USMLE

B Prader willi Syndrome

C Beckwith Widemann
Syndrome

D A& C

24.89% People got this right

Explanation:

Correct Answer (A) Angelmann Syndrome. Remember Prader Willi Syndrome


occurs due to Paternal deletion, paternal imprinting and maternal disomy.
Angelmann is exactly opposite of this. Beckwith Widemann syndrome is due to
loss of maternal imprinting or paternal disomy of IGF2 gene on chromosome 11.

Question: 56
Which of the following is not true for grade of tumor?

A It tells us extend of spread

B It tells prognosis

C Based on histology

D Chemotherapy is based on
grade

24.9% People got this right

Explanation:

Correct Answer (A) Stage tells extent of spread not grade. Grade tell the
differentiation of the tumor.

https://emedicoz.com/testresult/683510s14692521 39/149
6/9/24, 12:55 PM Neet PG Preparation, Neet PG Coaching, FMGE, USMLE

Question: 57
All the following are true about paroxysmal nocturnal hemoglobinuria
except?

A Inherited mutation of PIGA


gene

B Coombs test negative

C Thrombosis

D Thrombocytopenia

11.58% People got this right

Explanation:

Correct Answer (A) PNH is acquired condition not inherited.

Question: 58
A patient underwent gastrectomy 4 months back and has developed
anemia now. Likely diagnosis?

A Vitamin B12

B Iron

C Folic acid

D None of the above

23.79% People got this right

https://emedicoz.com/testresult/683510s14692521 40/149
6/9/24, 12:55 PM Neet PG Preparation, Neet PG Coaching, FMGE, USMLE

Explanation:

Correct Answer (B) Gastrectomy can lead to both iron and Vitami B12 deficiency.
But B12 deficiency takes 1-2 years to set in while iron deficiency sets early thus
IDA is answer.

Question: 59
Peripheral blood smear of patient presenting with early fatigue and fever is
as follows. What is likely diagnosis?

A Infectious mononucleosis

B B-Chronic lymphocytic
leukaemia

C Sezary syndrome

D Hairy cell leukaemia

62.76% People got this right

Explanation:

Correct Answer (D) Hairy cell leukaemia. Image shows cells with hair like
projections seen in HCL.

https://emedicoz.com/testresult/683510s14692521 41/149
6/9/24, 12:55 PM Neet PG Preparation, Neet PG Coaching, FMGE, USMLE

Question: 60
What is role of thrombomodulin in hemostasis?

A Activate fibrinogen

B Activate factor V

C Activate protein C

D Inhibit factor IX,X

31.05% People got this right

Explanation:

Correct Answer (C) Activate protein C. Thrombomodulin activates protein C


which inturn along with protein S induces lysis of factor V thus inhibiting
coagulation.

Question: 61
Histological image of Myocardial infarction shows coagulative necrosis
along with dense neutrophilic infiltrate. What is likely age of infarct?

A <1 day

B 1-3 days

C 5 days

D >1 week

52.96% People got this right

https://emedicoz.com/testresult/683510s14692521 42/149
6/9/24, 12:55 PM Neet PG Preparation, Neet PG Coaching, FMGE, USMLE

Explanation:

Correct Answer (B) 1-3 days. In < 1 days only necrosis is seen without
neutrophils.

Question: 62
Obstruction in emphysema is due to?

A Increased thickness of wall


due to glandular hypertrophy

B Collapse of terminal
bronchiole due to loss of elastin

C Dilated alveolar spaces due


to loss of elastin

D Destruction of cartilage
due to infection

33.11% People got this right

Explanation:

Correct Answer (B) In emphysema the obstruction is functional due to loss of


elastic recoil of terminal bronchioles. Option A is Chronic bronchitis option D is
bronchiectasis.

Question: 63
Sarcoma expressing cytokeratin is?

A Rhabdomyosarcoma

https://emedicoz.com/testresult/683510s14692521 43/149
6/9/24, 12:55 PM Neet PG Preparation, Neet PG Coaching, FMGE, USMLE

B Osteosarcoma

C Fibrosarcoma

D Synovial sarcoma

13.63% People got this right

Explanation:

Correct Answer (D) Synovial sarcoma. Synovial sarcoma is biphasic showing both
malignant mesenchymal and epithelial components, it shows positivity to CK
and also shows lymphatic spread.

Question: 64
ER+ PR+ Her2 Neu +++ tumors are classified as?

A Luminal A

B Luminal B

C Her 2 Neu over expressing

D Basal like

9.98% People got this right

Explanation:

Correct Answer (B)

Luminal B.

https://emedicoz.com/testresult/683510s14692521 44/149
6/9/24, 12:55 PM Neet PG Preparation, Neet PG Coaching, FMGE, USMLE

Question: 65
Which of the following does not have the histological image as shown on
biopsy?

A Minimal change disease

B Lupus nephritis

C Wegners Granulomatosis

D Good pastuer syndrome

50.96% People got this right

Explanation:

https://emedicoz.com/testresult/683510s14692521 45/149
6/9/24, 12:55 PM Neet PG Preparation, Neet PG Coaching, FMGE, USMLE

Correct Answer (A) Minimal change disease. The image shows cresents in
glomeruli and minimal doesnot present with cresentic GN.

Question: 66
A completely unimmunised aged 24 months comes to you at the
immunisation centre. Which of the following vaccines under the National
Immunization Schedule (NIS) won’t be given?

A BCG

B JE

C DPT

D MR

65.26% People got this right

Explanation:

Correct Answer (A)

BCG Explanation

• The concept of delayed vaccination is based on maximum age cutoffs of


vaccination.

• In the question as it is a 2-year-old child so all vaccines whose cutoff is less


than 1 years won’t be given to the child.

Vaccine Maximum Age

Group

BCG 1 year

https://emedicoz.com/testresult/683510s14692521 46/149
6/9/24, 12:55 PM Neet PG Preparation, Neet PG Coaching, FMGE, USMLE

Hep B 1 year (under

program)

IPV 1 year

Pentavalent 1 year

Rota 1 year

Measles 5 years

OPV 5 years

DPT 7 years

JE 15 years

TT/ dT No cut off

Question: 67
Which of the following vaccines doesn’t follow open vial policy; that can’t
be used after 4 hours of opening of vial?

A JE

B Rota

https://emedicoz.com/testresult/683510s14692521 47/149
6/9/24, 12:55 PM Neet PG Preparation, Neet PG Coaching, FMGE, USMLE

C IPV

D DPT

16.75% People got this right

Explanation:

Correct Answer (C)

IPV

Explanation

Open Vial Policy

Question: 68
In which coloured bag/ box would you discard a broken vaccine vial?

A White

B Blue

C Red

https://emedicoz.com/testresult/683510s14692521 48/149
6/9/24, 12:55 PM Neet PG Preparation, Neet PG Coaching, FMGE, USMLE

D Yellow

51.2% People got this right

Explanation:

Correct Answer (B)

Blue

Question: 69
Which is the program providing zero denial to patients at time of delivery
at government facilities?

A MusQan

B SUMAN

C SAPNA

D PMMVY

32.15% People got this right

Explanation:

Correct Answer (B)


https://emedicoz.com/testresult/683510s14692521 49/149
6/9/24, 12:55 PM Neet PG Preparation, Neet PG Coaching, FMGE, USMLE

SUMAN

Question: 70
Under NBSU number of beds available for paediatric care is?

A 5

B 10

C 20

D 30

11.9% People got this right

Explanation:

Correct Answer (C)

https://emedicoz.com/testresult/683510s14692521 50/149
6/9/24, 12:55 PM Neet PG Preparation, Neet PG Coaching, FMGE, USMLE

20

Question: 71
Which of the disease does have a 2nd attack?

A Mumps

B Measles

C Rubella

D Chicken Pox

39.64% People got this right

Explanation:

Correct Answer (D) Chicken Pox Explanation Classically we don’t report 2nd
attack from any of the 4 stated diseases as they provide lifelong immunity but
now, we are getting few cases of chicken pox in adults who had suffered in
childhood as well.

Question: 72

https://emedicoz.com/testresult/683510s14692521 51/149
6/9/24, 12:55 PM Neet PG Preparation, Neet PG Coaching, FMGE, USMLE

A pregnant female had got bitten by her own pet dog (a 11-month-old
puppy) 1 month back and had taken full course of rabies vaccination; has
not again got bitten by a monkey now. What is the line of management
now?

A Give ARV + Ig again.

B Give Ig again.

C Give ARV again.

D Do nothing.

51.44% People got this right

Explanation:

Correct Answer (D)

Do nothing.

Explanation

Nothing must be done as the duration of re- exposure is only one month.

Question: 73

https://emedicoz.com/testresult/683510s14692521 52/149
6/9/24, 12:55 PM Neet PG Preparation, Neet PG Coaching, FMGE, USMLE

Which component of the following options is not directly used to


calculate the Modified Kuppuswamy scale?

A Income of family

B Income of HOH

C Education of HOH

D Occupation of HOH

35.46% People got this right

Explanation:

Correct Answer (B)

Income of HOH

Question: 74
Mitigation in disaster cycle refers to:

A Triage

https://emedicoz.com/testresult/683510s14692521 53/149
6/9/24, 12:55 PM Neet PG Preparation, Neet PG Coaching, FMGE, USMLE

B Lowering the impact

C Mock drills

D Prompt management

50% People got this right

Explanation:

Correct Answer (B)

Lowering the Impact

Explanation

Mitigation means lowering the impact.

Question: 75
Identify the logo:

https://emedicoz.com/testresult/683510s14692521 54/149
6/9/24, 12:55 PM Neet PG Preparation, Neet PG Coaching, FMGE, USMLE

A Ujala

B Ujjwala

C Ujjawala

D Ujjaawala

54.8% People got this right

Explanation:

Correct Answer (B)

Ujjwala

https://emedicoz.com/testresult/683510s14692521 55/149
6/9/24, 12:55 PM Neet PG Preparation, Neet PG Coaching, FMGE, USMLE

Question: 76
Jan Arogya Yojna provides a health insurance of INR ____ lakhs / year to
BPL families in a year:

A 1

B 2.5

C 5

D 7.5

45.92% People got this right

Explanation:

Correct Answer (C) 5 Explanation PMJAY: • Cashless & paperless access (both
private & public) • States are free to choose the modalities for implementation
(implement through insurance company or directly through trust/ society or
mixed level) • Benefits are portable over the country. • Increased benefit cover to
nearly 40% of the population • Covering almost all secondary and many tertiary
hospitalizations (except a negative list) - 1,393 procedures • Coverage of 5 lakh
for each family (no restriction of family size, age, or gender) • All pre–existing
conditions are covered. • Covers cost of hospitalization, treatment, up to 3 days
of pre-hospitalization and 15 days of posthospitalization follow-up care

Question: 77
Calculate the TFR for this population:

https://emedicoz.com/testresult/683510s14692521 56/149
6/9/24, 12:55 PM Neet PG Preparation, Neet PG Coaching, FMGE, USMLE

A 0.25

B 0.5

C 0.75

D 1.0

14.73% People got this right

Explanation:

Correct Answer (B) 0.5 Explanation TFR = (Sum total of ASFR * Interval in Each
Age Group)/ 1000 TFR = (100 * 5)/ 1000 = 0.5

Question: 78
Which of the following disease is not spread by sandfly as a vector?

A Chandipura encephalitis

B Oraya fever

C Sandfly fever

https://emedicoz.com/testresult/683510s14692521 57/149
6/9/24, 12:55 PM Neet PG Preparation, Neet PG Coaching, FMGE, USMLE

D White socks fever

35.8% People got this right

Explanation:

Correct Answer (D)

White socks fever

Question: 79
Which is not true for Focussed Group Discussion (FGD)?

A 6- 12 participants.

B Should know each other


from before.

C Should discuss on a
common issue.

D Can only be done in


educated participants.

26.92% People got this right

https://emedicoz.com/testresult/683510s14692521 58/149
6/9/24, 12:55 PM Neet PG Preparation, Neet PG Coaching, FMGE, USMLE

Explanation:

Correct Answer (D) Can only be done in educated participants Explanation


Focussed Group Discussion: • 6- 12 members • 1 moderator • Discuss on 1 issue.
• Disadvantage: unequal/ useless discussion

Question: 80
Survival rate is used of all except:

A Cancers

B Road traffic accidents

C Common cold

D Zaire variant of Ebola

62.48% People got this right

Explanation:

Correct Answer (C) Common cold Explanation Common cold has no mortality so
SR or CFR will have least use for it

Question: 81
This clinical examination is not useful in which of the following conditions

https://emedicoz.com/testresult/683510s14692521 59/149
6/9/24, 12:55 PM Neet PG Preparation, Neet PG Coaching, FMGE, USMLE

A Thyroid eye disease

B Carotico-cavernous fstula

C Entropion

D Lacrimal gland tumour

37.86% People got this right

Explanation:

Correct Answer (C) Clinical evaluation of proptosis by Nafzigger view Entropion


does not cause proptosis

Question: 82
A 2yr old child brought by the parents with complaints of enlarged eyes.
There was no other complaint. IOP was normal. Cornea was clear. Rest of
eye examination was also normal. Most probable diagnosis is?

A Congenital glaucoma

B Retinoblastoma

https://emedicoz.com/testresult/683510s14692521 60/149
6/9/24, 12:55 PM Neet PG Preparation, Neet PG Coaching, FMGE, USMLE

C Congenital endothelial
dystrophy

D Megalocornea

53.17% People got this right

Explanation:

Correct Answer (D)

IOP normal rules out congenital glaucoma Cornea clear rules of endothelial
dystrophy Eye examination was norma, so retinoblastoma ruled out

Question: 83
These spectacles are useful in following except

A Myopic astigmatism in 16
year old

B Accommodative esotropia
in 2 year old child

C Pediatric pseudophakia of
1 year old child

https://emedicoz.com/testresult/683510s14692521 61/149
6/9/24, 12:55 PM Neet PG Preparation, Neet PG Coaching, FMGE, USMLE

D Presbyopia in 50 year old

30.85% People got this right

Explanation:

Correct Answer (A) Bifocal spectacles (executive type) Myopic astigmatism does
not need bifocals It requires spherical and cylinder lenses

Question: 84
A patient presents with abnormal facies, frontal baldness and with weakness
of the hands and difficulty in walking. He has difficulty in releasing grip,
muscle wasting and weakness. Ocular signs include all except?

A Ptosis

B Pigmentory retinopathy

C Subluxated lens

D Christmas tree cataract

5.57% People got this right

Explanation:

https://emedicoz.com/testresult/683510s14692521 62/149
6/9/24, 12:55 PM Neet PG Preparation, Neet PG Coaching, FMGE, USMLE

Correct Answer (C)

Myotonic dystrophy

Not associated with subluxation of lens

Question: 85
Identify

A Vossius ring

B Pseudoexfoliation

C Foreign body

D Ocular trauma

39.97% People got this right

Explanation:

Correct Answer (B)

Psudoexfoliation material on lens

https://emedicoz.com/testresult/683510s14692521 63/149
6/9/24, 12:55 PM Neet PG Preparation, Neet PG Coaching, FMGE, USMLE

Question: 86
Where is the likely lesion here

A Left Optic tract

B Left Occipital lobe

C Right Occipital lobe

D Optic chiasma

24.95% People got this right

Explanation:

Correct Answer (B)

Right homonymous hemianopia with macula sparing

Question: 87
What could be the presenting symptom of a person with diabetic cystoid
macular edema

A Sudden painless loss of

https://emedicoz.com/testresult/683510s14692521 64/149
6/9/24, 12:55 PM Neet PG Preparation, Neet PG Coaching, FMGE, USMLE

vision with metamorphosia

B Sudden painful loss of


vision with floaters

C Gradual painless loss of


vision with metamorphosia

D Gradual painful loss of


vision with metamorphosia

45.68% People got this right

Explanation:

Correct Answer (C) Macular edema will present with gradual painless vision loss
with metamorphosia as central retina is involved

Question: 88
-4.00DS with -1.00DC ×180 degrees. What is false?

A Vertical axis will be steeper

B Compound myopic
astigmatism with the rule

C During cataract surgery of


this patient, incision in horizontal
axis is preferred

D Spherical equivalent of this


will be -4.5 DS

18.95% People got this right

https://emedicoz.com/testresult/683510s14692521 65/149
6/9/24, 12:55 PM Neet PG Preparation, Neet PG Coaching, FMGE, USMLE

Explanation:

Correct Answer (C) After conversion to cross formulation It will be -5 D in vertical


-4D in horizontal During cataract surgery,incision will flatter the axis, so in this
question, incision should be given in vertical axis

Question: 89
What will be the result if cold water is poured in right ear of the patient

A Fast phase of nystagmus to


the right side

B Fast phase of nystagmus to


the left side

C Slow phase of nystagmus


to the left side

D No nystagmus

57.87% People got this right

Explanation:

Correct Answer (B) COWS Cold water in right ear induces left nystagmus left fast
and slow slow phase

Question: 90
What will be the pupil reaction if you shine light in right eye of this patient?

https://emedicoz.com/testresult/683510s14692521 66/149
6/9/24, 12:55 PM Neet PG Preparation, Neet PG Coaching, FMGE, USMLE

A Right pupil constrict, left


doesn’t

B Both pupil constrict

C Neither pupil constrict

D Left pupil constrict, right


doesn’t

32.05% People got this right

Explanation:

Correct Answer (A) Left hypo and exotropia with ptosis Left 3rd nerve palsy Right
pupil will constrict but left doesn’t when we shine light in right eye

Question: 91
A 9-year-old boy is brought to the clinic by his mother for evaluation of
rash and sore throat. Two days ago, the patient returned home from
summer camp with a fever, sore throat, and rash on his extremities as
shown in the image. The fever has resolved, but the rash has continued to
spread. The patient has no chronic medical conditions and has received all
routine vaccinations. Vital signs are normal. Oropharyngeal examination
shows superficial ulcers on the buccal mucosa and soft palate but no
tonsillar enlargement or exudates. The infectious agent most likely
https://emedicoz.com/testresult/683510s14692521 67/149
6/9/24, 12:55 PM Neet PG Preparation, Neet PG Coaching, FMGE, USMLE

responsible for this patient's condition is also associated with which of the
following complications?

A Facial nerve palsy

B Keratitis

C Myocarditis

D Symmetric polyarthritis

35.85% People got this right

Explanation:

Correct Answer (C) This patient has characteristic findings of hand-foot-and-


mouth disease (HFMD), an infection caused by Coxsackievirus. HFMD is
characterized by a painful oral enanthem consisting of vesicles/ulcers usually
affecting the tongue, buccal mucosa, or soft palate as well as a maculopapular or
vesicular exanthem on the hands and feet (including palms and soles). Illness
may be accompanied by a fever and is usually self-limited. Herpangina: This
condition is not associated with an exanthem but causes high fever and a painful
oral enanthem primarily affecting the posterior oropharynx and sparing the
gingiva. Myocarditis: Patients often present with a viral prodrome (eg, fever,
myalgia) followed by signs of heart failure, such as dyspnea, tachypnea, and
tachycardia. Myocarditis may also occur as a rare complication of HFMD due to
widespread viremia prior to the development of protective antibodies.

https://emedicoz.com/testresult/683510s14692521 68/149
6/9/24, 12:55 PM Neet PG Preparation, Neet PG Coaching, FMGE, USMLE

Question: 92
A 25-year-old primigravida has a stillbirth at 18 weeks gestation. Her only
symptom during pregnancy was pain in both knees and feet, which she
attributed to pregnancy-related weight gain and being "on my feet all
day" as an elementary school teacher. The pain lasted approximately a
week and resolved without medication. The patient was taking prenatal
vitamins daily, and her prenatal care was appropriate. Fetal autopsy
shows pleural effusion pulmonary hypoplasia, and ascites. Infection with
which of the following is the most likely etiology of the stillbirth?

A Enveloped, double-
stranded DNA virus

B Enveloped, single-stranded
RNA virus

C Gram-positive coccus in
chains

D Nonenveloped, single-
stranded DNA virus

17.35% People got this right

Explanation:

Correct Answer (D)

Parvovirus is a nonenveloped single-stranded (ss) DNA virus with an increased


incidence in children and teachers, as most individuals are infected during
school outbreaks.
Infection in adults can present with an acute, symmetric arthralgia/arthritis
involving hands, wrists, knees, and/or feet, with or without rash.
Fetal infection with parvovirus can lead to interruption of erythropoiesis, causing
profound anemia and congestive heart failure.
Fetal congestive heart failure can cause pleural effusions, pericardial effusions,
and ascites.
The findings on fetal autopsy (pleural effusion with secondary pulmonary
hypoplasia, and ascites) represent fetal hydrops.

https://emedicoz.com/testresult/683510s14692521 69/149
6/9/24, 12:55 PM Neet PG Preparation, Neet PG Coaching, FMGE, USMLE

Parvovirus is also responsible for erythema infectiosum ("fifth disease"), which


presents with a "slapped cheek" appearance in children, and aplastic crisis in
patients with sickle cell anemia.

Question: 93
A 43-year-old man undergoes induction chemotherapy for acute myeloid
leukemia. Three weeks later, he is hospitalized with fever, cough, and
generalized weakness. On physical examination, crackles are heard over the
right upper lung. Chest x-ray reveals a dense infiltrate involving the right
upper lung lobe. Broad-spectrum antibiotic therapy is instituted. Five days
later, the patient is still febrile. Bronchoscopy with biopsy is performed and
the tissue samples from the bronchoscopy grow mold as shown in the
image. Which of the following organisms is most likely responsible for this
patient's current condition?

A Aspergillus fumigatus

B Candida albicans

C Cryptococcus neoformans

D Pneumocystis jirovecii

62.76% People got this right

https://emedicoz.com/testresult/683510s14692521 70/149
6/9/24, 12:55 PM Neet PG Preparation, Neet PG Coaching, FMGE, USMLE

Explanation:

Correct Answer (A)

This patient with leukemia recently underwent chemotherapy (with likely


myelosuppressive effects) and now has a fungal respiratory infection with febrile
neutropenia (FN), typically defined as fever with an absolute neutrophil count
<500/mm3.
Patients with FN are at increased risk for severe infection.
Broad-spectrum antibiotic therapy is indicated. Gram positive bacteria are the
most identified pathogens in FN followed by gram-negative organisms.
However, patients with profound and prolonged neutropenia (several days) are
at especially high risk for fungal infections.
Invasive pulmonary aspergillosis presents with some combination of fever, chest
pain, cough, dyspnea, and hemoptysis.
On light microscopy, Aspergillus is seen as septate narrow hyphae that branch at
a 45° angle.
Antifungal therapy is needed.

Question: 94
All the following organisms are positive for the test shown in the image
except

A Cryptococcus neoformans

B Brucella abortus

C Campylobacter jejuni

https://emedicoz.com/testresult/683510s14692521 71/149
6/9/24, 12:55 PM Neet PG Preparation, Neet PG Coaching, FMGE, USMLE

D Helicobacter pylori

2.16% People got this right

Explanation:

Correct Answer (C)

Image shown here is Urease test.


It is positive for Proteus, Ureaplasma, Brucella, H.pylori and Nocardia

Question: 95
A 4-year-old male child is evaluated in your clinic for recurrent skin and
respiratory infections. He has light skin and silvery hair Horizontal
nystagmus is present on eye examination. Peripheral blood smear
demonstrates giant cytoplasmic granules in neutrophils and monocytes.
The patient most likely suffers from which of the following disorders?

A Chronic granulomatous
disease

B DiGeorge syndrome

C Wiskott-Aldrich syndrome

D Chediak-Higashi syndrome

34.65% People got this right

Explanation:

Correct Answer (D)

The patient described in this question suffers from Chediak-Higashi syndrome.


Chediak-Higashi syndrome is an autosomal recessive syndrome consisting of
immunodeficiency, albinism and neurologic defects that is most commonly
diagnosed in childhood.

https://emedicoz.com/testresult/683510s14692521 72/149
6/9/24, 12:55 PM Neet PG Preparation, Neet PG Coaching, FMGE, USMLE

Neurologic defects associated with this condition include nystagmus (as


mentioned in the question stem), as well as peripheral and cranial neuropathies,
The immunodeficiency in Chédiak-Higashi syndrome results from a defect in
neutrophil phagosome lysosome fusion.
This causes abnormal giant lysosomal inclusions that are visible on light
microscopy of a peripheral blood smear.
The immunodeficiency leads to recurrent pyogenic infections most commonly
caused by Staphylococci and Streptococci.
Abnormal melanin storage in melanocytes causes partial oculocutaneous
albinism.

Question: 96
A 24-year-old woman comes to the emergency department due to 2 days
of fever. The patient has no chronic medical conditions. drinks alcohol
occasionally. and does not use tobacco or illicit drugs. She is sexually active
and frequently travels for her work as a freelance journalist. Physical
examination is notable for mild scleral icterus and splenomegaly. Laboratory
findings are shown in the image. Which of the following infectious agents is
most likely responsible for this patient ‘s current condition?

A Plasmodium falciparum

B Schistosoma mansoni

C Toxocara canis

https://emedicoz.com/testresult/683510s14692521 73/149
6/9/24, 12:55 PM Neet PG Preparation, Neet PG Coaching, FMGE, USMLE

D Trichinella spiralis

53.77% People got this right

Explanation:

Correct Answer (A) This patient with fever. Icterus, splenomegaly and ring-
shaped parasites within red blood cells has malaria, which is caused by 4 main
species of Plasmodium {P falciparum, P vivax. P ovale. P malaria). The parasite is
transmitted by female Anopheles mosquitos during blood feeding. travels to the
liver where it infects hepatocytes. and then subsequently undergoes asexual
reproduction, which generates hepatic schizonts with thousands of daughter
cells (merozoites). The diagnosis is confirmed when peripheral blood microscopy
shows intraerythrocytic trophozoites with a "diamond ring" appearance.

Question: 97
Vector for Trypanosoma cruzi is

A Tsetse fly

B Triatome bug

C Phlebotomus fly

D Simulium fly

29.26% People got this right

Explanation:

Correct Answer (B)

Vector for T. cruzi is Triatome bug (Kissing, cone nose, assassin and reduviid bug)
Tsetse fly – T. brucei
Phlebotomus fly – Leishmania
Simulium fly – Onchocerca volvulus

https://emedicoz.com/testresult/683510s14692521 74/149
6/9/24, 12:55 PM Neet PG Preparation, Neet PG Coaching, FMGE, USMLE

Question: 98
A 10-year-old boy is brought to the emergency department alter
experiencing high fevers and chills for the last few days. He also
complains of dull pain just above his left knee. He has no history of
recent trauma other than minor scrapes to his knees and elbows while
playing outside. Physical examination shows point tenderness 3 cm above
the kneecap. There is no joint effusion. Radiographs show soft-tissue
swelling and a periosteal reaction over the lower end of the femur. Which
of the following organisms is most likely responsible for this patient's
symptoms?

A Enterococcus faecalis

B Staphylococcus aureus

C Staphylococcus
epidermidis

D Pseudomonas aeruginosa

59.92% People got this right

Explanation:

Correct Answer (B) Osteomyelitis is an infection of bone and bone marrow that
occurs by 1 of 3 mechanisms: Hematogenous seeding due to an episode of
bacteremia Spread from a contiguous focus of infection. as occurs in an infected
diabetic foot wound Direct inoculation of bone, such as with a compound
fracture Hematogenous osteomyelitis occurs predominantly in children
[particularly boys) and most frequently affects the long bones. The tibia, fibula
and femur are most often involved. Adults who develop the condition are more
likely to have vertebral involvement and frequently have a predisposition to
bacteraemia due to risk factors such as IV drug abuse or indwelling vascular
catheters. Overall MCC is Staphylococcus aureus

Question: 99
https://emedicoz.com/testresult/683510s14692521 75/149
6/9/24, 12:55 PM Neet PG Preparation, Neet PG Coaching, FMGE, USMLE

A 34-year-old woman presented with 3 days of discomfort and swelling in


the right axilla. She owns a parrot and a hamster. Temperature is 38.9 C.
On physical examination. There is an enlarged. Tender axillary lymph
node on the right measuring about 3 cm and with slight surrounding
erythema but no skin breakdown. Several scratch marks are seen on the
right arm. The organism responsible for the lymphadenopathy is also
associated with which of the following conditions?

A Bacillary angiomatosis

B Bladder cancer

C Condyloma acuminata

D Hemolytic uremic
syndrome

39.69% People got this right

Explanation:

Correct Answer (A) B. henselae causes Cat-scratch disease (CSD) which is an


infectious disease that results from a scratch or bite of a cat. Symptoms typically
include a non-painful bump or blister at the site of injury and painful and
swollen lymph nodes. People may feel tired, have a headache, or a fever.
Symptoms typically begin within 3-14 days following infection.

Question: 100
Kanagawa phenomenon in Wagatsuma agar is exhibited by

A Vibrio cholerae

B Vibrio parahaemolyticus

https://emedicoz.com/testresult/683510s14692521 76/149
6/9/24, 12:55 PM Neet PG Preparation, Neet PG Coaching, FMGE, USMLE

C Vibrio vulnificus

D Vibrio alginolyticus

46.52% People got this right

Explanation:

Correct Answer (B)

The pathogenic strains of V. parahaemolyticus when grown on a special high-


salt mannitol blood agar medium named as Wagatsuma agar show hemolysis.
This hemolysis is caused by a thermostable hemotoxin released by pathogenic
strains, which are known as Kanagawa-positive strains.
Nonpathogenic strains isolated from environment that do not cause any
hemolysis on the blood agar are called Kanagawa-negative strains.

Question: 101
A patient on Sofosbuvir should periodically estimate

A RFT

B LFT

C ECG

D EEG

8.27% People got this right

Explanation:

Correct Answer (C)

Sofosbuvir- antiviral drug, useful in chronic hepatitis C virus infection, causes


adverse effects of cardiotoxicity - bradycardia and heart block, so it needs ECG

https://emedicoz.com/testresult/683510s14692521 77/149
6/9/24, 12:55 PM Neet PG Preparation, Neet PG Coaching, FMGE, USMLE

monitoring every week for at least first three months

Question: 102
All of the followings are matched correctly, EXCEPT

A A

B B

C C

D D

9.47% People got this right

Explanation:

Correct Answer (B) Nalmefene nasal spray approved for treating opioid
overdose, not for dependence Sotagliflozin---- SGLT-2 inhibitor useful for heart
failure Fezolinetant – NK3 antagonist, useful for vasomotor symptoms associated
with menopause Trofinetide – approved for Rett syndrome

Question: 103
Which one of the following medications inhibits fibrinolysis?

https://emedicoz.com/testresult/683510s14692521 78/149
6/9/24, 12:55 PM Neet PG Preparation, Neet PG Coaching, FMGE, USMLE

A Aminocaproic acid

B Heparin

C Lepirudin

D Lepirudin

42.38% People got this right

Explanation:

Correct Answer (A) Fibrinolytic drugs ( streptokinase, urokinase, alteplase,


Tenecteplase) causes adverse effects of bleeding. Antidote for thrombolytic
agents are epsilon aminocaproic acid.

Question: 104
Which of the following antidepressant drug is contraindicated in patients
with history of seizure?

A Fluoxetine

B Bupropion

C Venlafaxine

D Mirtazapine

26.91% People got this right

Explanation:

https://emedicoz.com/testresult/683510s14692521 79/149
6/9/24, 12:55 PM Neet PG Preparation, Neet PG Coaching, FMGE, USMLE

Correct Answer (B) Bupropion causes adverse effects of seizure and anxiety,
hence contraindicated in epileptic patients

Question: 105
Match the following

A 1-B, 2-D, 3-A, 4-C

B 1-D, 2-B, 3-A, 4-C

C 1-B, 2-C, 3-A, 4-D

D 1-C, 2-D, 3-A, 4-B

24.65% People got this right

Explanation:

Correct Answer (A) Droxidopa a prodrug of dopamine useful for managing


dialysis induced hypotension Lofexidine an alpha 2 agonist approved for
managing opioid withdrawal symptoms Dexmedetomidine an alpha 2 agonist
useful for preanaesthetic medication, sedation of ICU patients and sublingual
film formulation recently approved for psychosis and bipolar disorder Clonidine
an alpha 2 agonist can be useful for prophylaxis of chronic migraine

https://emedicoz.com/testresult/683510s14692521 80/149
6/9/24, 12:55 PM Neet PG Preparation, Neet PG Coaching, FMGE, USMLE

Question: 106
A 14-year-old boy returns from a Boy Scout backpack trip with foul-
smelling watery diarrhea. On further questioning, he admits to drinking
water from a mountain brook without first boiling it. Stool is sent for ova
and parasites, confirming the diagnosis of Giardia lamblia infection.
Which of the following drugs is appropriate treatment?

A Metronidazole

B Nifurtimox

C Suramin

D Mebendazole

53.48% People got this right

Explanation:

Correct Answer (A) Metronidazole is used to treat protozoal infections due to


Giardia, Entamoeba, and Trichomonas spp. Nifurtimox is used to treat Chagas
disease (due to Trypanosoma cruzii). Suramin is used to treat African
trypanosomiasis. Mebendazole is used to treat round worm infections

Question: 107
Which of the following drugs is correctly associated with its clinical
application?

https://emedicoz.com/testresult/683510s14692521 81/149
6/9/24, 12:55 PM Neet PG Preparation, Neet PG Coaching, FMGE, USMLE

A A

B B

C C

D D

51.75% People got this right

Explanation:

Correct Answer (D) Ferrous sulfate – useful for microcytic anemia Iron dextran
useful in iron deficiency anemia, for macrocytic anemia supplement VIT B12 and
folic acid Filgrastim is the recombinant granulocyte colony stimulating factor
useful for neutropenia not for thrombocytopenia, for thrombocytopenia due to
cancer chemotherapy orelvekin (IL-11) is used Eltrombopag – useful in
thrombocytopenia

Question: 108
A pregnant female with history of asthma presented in third stage of
labour. Which drug is to be avoided in her?

A Carboprost

https://emedicoz.com/testresult/683510s14692521 82/149
6/9/24, 12:55 PM Neet PG Preparation, Neet PG Coaching, FMGE, USMLE

B Misoprostol

C Oxytocin

D Dinoprostone

48.82% People got this right

Explanation:

Correct Answer (A) Carboprost may cause bronchospasm, contra indicated in


asthma patients

Question: 109
Which one of the following is a fully human monoclonal antibody?

A Rituximab

B Bevacizumab

C Adalimumab

D Tositumomab

28.06% People got this right

Explanation:

Correct Answer (C) Adalim u mab- U- fully human Tositum o mab- fully mouse
Ritu xi mab- chimerical Bevaci zu mab – humanized

Question: 110
Which of the following is an action of a non-competitive antagonist?
https://emedicoz.com/testresult/683510s14692521 83/149
6/9/24, 12:55 PM Neet PG Preparation, Neet PG Coaching, FMGE, USMLE

A Alters the mechanism of


action of an agonist

B Alters the potency of an


agonist

C Shifts the dose–response


curve of an agonist to the right

D Decreases the maximum


response to an agonist

32% People got this right

Explanation:

Correct Answer (D) A non-competitive antagonist decreases the magnitude of


the response to an agonist but does not alter the agonist’s potency (i.e., the
ED50 remains unchanged).

Question: 111
A fetus was found to have congenital heart block. What should be
evaluated in the mother?

A APLA Syndrome

B Congenital heart defects

C SLE

D Hemolytic anemia

4.04% People got this right

https://emedicoz.com/testresult/683510s14692521 84/149
6/9/24, 12:55 PM Neet PG Preparation, Neet PG Coaching, FMGE, USMLE

Explanation:

Correct Answer (B) Newborn with congenital heart block, born to mothers with
Sjogren's syndrome and SLE have these issues due to anti Ro/SSA antibodies
from mother

Question: 112
A child presents with high grade fever, inspiratory stridor and develops
swallowing difficulty and drooling of saliva within 4-6 hours. Along with
airway management, which of the following needs to be given?

A Nebulized Racemic
epinephrine

B Anti-diphtheria toxin

C
Corticosteroids/Conservative

D IV Ceftriaxone

23.45% People got this right

Explanation:

Correct Answer (D) Suspect Epiglottitis/ Laryngeal diphtheria. For epiglottitis,


airway and iv antibiotics (ceftriaxone) are a must.

Question: 113
In Cystic Fibrosis, the mutation affects which ion transport, which amino
acid, and is at which position?

A 508; phenylalanine; Cl

https://emedicoz.com/testresult/683510s14692521 85/149
6/9/24, 12:55 PM Neet PG Preparation, Neet PG Coaching, FMGE, USMLE

B 708; tryptophan; Ca

C 708; tryptophan; Cl

D 708; phenylalanine; Cl

39.64% People got this right

Explanation:

Correct Answer (A) DeltaF 508 is the most common mutation in cystic fibrosis in
caucasions and indians.

Question: 114
Regarding UTI in children, correct options

A Most common is
S.pneumoniae

B Bladder and bowel


dysfunction increases the risk

C Recurrent UTI should get


MCU

D 5 yrs old febrile UTI, needs


MCU

18.26% People got this right

Explanation:

Correct Answer (B) In girls of 5-10 yrs, bowel bladder dysfunction adds to the
VUR and leads to recurrent UTI

https://emedicoz.com/testresult/683510s14692521 86/149
6/9/24, 12:55 PM Neet PG Preparation, Neet PG Coaching, FMGE, USMLE

Question: 115
Not Useful in the management of GBS

A Plasmapheresis

B IV Immunoglobulins

C Steroids

D Ventilatory Support

48.92% People got this right

Explanation:

Correct Answer (C) Steroids have no role in GBS/AIDP.

Question: 116
A 3 week neonate presents with ambiguous genitalia with Na+ - 127, K+ -
5.6meq, along with IV fluids, what specific management is done?

A Calcium gluconate

B Potassium resins /
sequestration

C Hydrocortisone iv

D Broad spectrum antibiotics

43.73% People got this right

Explanation:
https://emedicoz.com/testresult/683510s14692521 87/149
6/9/24, 12:55 PM Neet PG Preparation, Neet PG Coaching, FMGE, USMLE

Correct Answer (C) iv hydrocortisone is drug of choice for acute adrenal


insufficiency/ salt wasting crisis in CAH

Question: 117
What is the most common cause of CAH in children?

A 21 hydroxylase deficiency

B 11beta hydroxylase
deficiency

C 17 alpha hydroxylase
deficiency

D 3beta HSD deficiency

59.49% People got this right

Explanation:

Correct Answer (A) 90% CAH is caused by 21 hydroxylase deficiency

Question: 118
Age at which handedness develops?

A 2 years

B 3 years

C 4 years

https://emedicoz.com/testresult/683510s14692521 88/149
6/9/24, 12:55 PM Neet PG Preparation, Neet PG Coaching, FMGE, USMLE

D 5 years

37.43% People got this right

Explanation:

Correct Answer (B) At 3 yrs handedness develops always

Question: 119
A 10 yrs old child with generalized edema has urine protein 3+ & serum
cholesterol 238mg/dl. His urine examination shows oval fat bodies What
is the most likely diagnosis?

A Nephrotic syndrome

B Nephritic syndrome

C Goodpasture syndrome

D Urinary tract infection

66.94% People got this right

Explanation:

Correct Answer (A) This is classic nephrotic syndrome

Question: 120
What is the first and best treatment for a frequent relapser nephrotic
syndrome?

https://emedicoz.com/testresult/683510s14692521 89/149
6/9/24, 12:55 PM Neet PG Preparation, Neet PG Coaching, FMGE, USMLE

A Low dose alternate day


prednisolone

B Levamisole

C MMF

D Cyclophosphamide

33.69% People got this right

Explanation:

Correct Answer (A) First and best step for frequent relapser or steroid dependent
nephrotic is low dose, long term alternate day prednisolone 0.5-0.7mg/kg/day

Question: 121
A patient of head injury has bluish discoloration over mastoid area,
bleeding from EAC and complaining of hearing loss with normal facial
nerve function. After three days, patient starts having facial weakness.
What is the possible diagnosis?

A Longitudinal fracture of
temporal bone

B Transverse fracture of
temporal bone

C Traumatic perilymphatic
fistula

D Traumatic fracture of
parietal bone

https://emedicoz.com/testresult/683510s14692521 90/149
6/9/24, 12:55 PM Neet PG Preparation, Neet PG Coaching, FMGE, USMLE

23.45% People got this right

Explanation:

Correct Answer (A) Longitudinal fracture of temporal bone Ref: Read the text
below Sol: • This is a case of longitudinal fracture of temporal bone. The ear
bleeding is a feature of longitudinal fracture. This fracture is due to fall on the
side. Battle sign is ecchymosis around the mastoid tip area which is seen in
temporal bone fractures. This fracture causes ossicular dislocation and hence
conductive hearing loss. Transverse fractures lead to cochlear injury and hence
cause SNHL. CSF otorrhoea is more common with longitudinal fractures • In
temporal bone fractures, the facial palsy is either of immediate (Transverse
fractures) or delayed onset ( Longitudinal fractures) • Immediate onset need
immediate surgery which is facial nerve decompression and if need be end to
end anastomosis or nerve grafting • Delayed onset should be managed with 3
weeks of steroid therapy as the cause is oedema in this case. If there is no
recovery after 3 weeks of steroid therapy, electrophysiological nerve testing
should be done (Electroneuronography). If there is significant facial nerve
degeneration, patient should be taken for surgery (facial nerve decompression)
should be done.

Question: 122
A 5 year old patient presents with the history of hearing loss. On
examination he is found to have a chalky white mass behind the intact
tympanic membrane as shown in the image . What is the most possible
diagnosis?

A Congenital cholesteatoma

https://emedicoz.com/testresult/683510s14692521 91/149
6/9/24, 12:55 PM Neet PG Preparation, Neet PG Coaching, FMGE, USMLE

B Otomycosis

C Otosclerosis

D Exosteosis

61.99% People got this right

Explanation:

Correct Answer (A) Congenital cholesteatoma Ref: Read the text below. Sol:- •
Congenital cholesteatomas are believed to arise from embryonal inclusions or
rests of epithelial cells. • Levenson, established a set of criteria for the definition
of congenital cholesteatoma in the middle ear. • These criteria included, (1) a
white mass medial to a normal tympanic membrane, (2) a normal pars flaccida
and pars tensa, (3) no prior otologic procedures. In addition, prior bouts of otitis
media were not grounds for exclusion.

Question: 123
A 20 year old male patient has high pitch voice. His fiberoptic
laryngoscopy is normal. The clinician has advised him for speech therapy
in which he is asked to pho-nate while pulling the thyroid cartilage
downwards and pressing it backwards. What is the name of this
manoeuvre?

A Heimlich manoeuvre.

B Gutzmann’s manoeuvre.

C Muller’s manoeuvre.

D Valsalva manoeuvre.

15.28% People got this right

https://emedicoz.com/testresult/683510s14692521 92/149
6/9/24, 12:55 PM Neet PG Preparation, Neet PG Coaching, FMGE, USMLE

Explanation:

Correct Answer (B)

Gutzmann’s manoeuvre.

Ref: Read the text below.

Sol:- Gutzmann’s manoeuvre is used in the management of Puberphonia. If


patient is not relieved with speech therapy, then Type III thyroplasty is done.

Question: 124
What type of tympanogram is given in the picture?

A Type A curve.

B Type B Curve.

https://emedicoz.com/testresult/683510s14692521 93/149
6/9/24, 12:55 PM Neet PG Preparation, Neet PG Coaching, FMGE, USMLE

C Type C Curve.

D Type AD curve.

38.3% People got this right

Explanation:

Correct Answer (B)

Type B Curve.

Ref: Read the text below.

Question: 125
Arrange the auditory pathway in order from periphery to centre A. Eight
nerve B. Inferior colliculus C. Medial geniculate body D. Ventral / dorsal
cochlear nucleus E. Superior Olivary nucleus F. Lateral lemniscus G.
Auditory cortex

A A, D, E, F, B, C, G

B B, C, G, A, D, E, F

C E, F, B, A, D, C, G

https://emedicoz.com/testresult/683510s14692521 94/149
6/9/24, 12:55 PM Neet PG Preparation, Neet PG Coaching, FMGE, USMLE

D A, D, E, F, C, G, B

16.15% People got this right

Explanation:

Correct Answer (A)

A, D, E, F, B, C, G

Ref: Read the text below.

Sol:- Auditory pathway--ECOLI MA E - Eight nerve C- Ventral / dorsal cochlear


nucleus O - Superior Olivary nucleus L - Lateral lemniscus I - Inferior colliculus M
- Medial geniculate body A - Auditory cortex

Question: 126
Which of the following is the most commonly used adjuvant for recurrent
respiratory papillomatosis (RRP) among members of American Society of
Pediatric Otolaryngology (ASPO)?

A Bevacizumab (Avastin)

B Intralesional cidofovir
(Vistide)

C Celecoxib (Celebrex)

https://emedicoz.com/testresult/683510s14692521 95/149
6/9/24, 12:55 PM Neet PG Preparation, Neet PG Coaching, FMGE, USMLE

D Interferon

32.48% People got this right

Explanation:

Correct Answer (B) Intralesional cidofovir (Vistide) Ref:Read the text below Sol •
Specific recommendations have been made by the Task Force on RRP from
ASPO in regard to the use of cidofovir. • Informed consent must be obtained
from the patient’s parents because this drug is being used in an off-label setting.

Question: 127
A Child underwent tonsillectomy and after surgery shifted to recovery
room where he started bleeding and on examination a clot was seen at
the site of surgery. Next step in management will be ?

A Blood transfusion only

B Shift patient to OT and


ligation bleeding vessel

C Shift patient to OT and


cautery.

D Use hemostatic agents do


3 to 4 gargles per day

15.09% People got this right

Explanation:

Correct Answer (B)

Shift patient to OT and ligation bleeding vessel

Ref: Read the text below

https://emedicoz.com/testresult/683510s14692521 96/149
6/9/24, 12:55 PM Neet PG Preparation, Neet PG Coaching, FMGE, USMLE

Sol: Primary haemorrhage.

• Occurs at the time of operation.

• It can be controlled by pressure, ligation or electrocoagulation of the bleeding


vessels. Reactionary haemorrhage.

• Occurs within a period of 24 h and can be controlled by simple measures such


as removal of the clot, application of pressure or vasoconstrictor.

• Presence of a clot prevents the clipping action of the superior constrictor


muscle on the vessels which pass through it (compare postpartum uterine
bleeding). Secondary haemorrhage.

• Usually seen between the fifth to tenth postoperative day.

• It is the result of sepsis and premature separation of the membrane.

Question: 128
Which of the following statements is true about inverted papilloma
arising from the lateral wall of nose?

A Polypoidal masses
resemble allergic nasal polyps

B Affects females in
adolescent period

C Sends early distant


metastases

D Treatment requires
maxillectomy with postoperative
radiotherapy

35.75% People got this right

https://emedicoz.com/testresult/683510s14692521 97/149
6/9/24, 12:55 PM Neet PG Preparation, Neet PG Coaching, FMGE, USMLE

Explanation:

Correct Answer (A) Polypoidal masses resemble allergic nasal polyps Sol : •
Inverted papilloma, which arises from the mucosa of lateral wall of nose, most
frequently affects males in the age group of 30-5O years. • Only 10-15% are
associated with malignancy. • Treatment is complete excision which amounts to
medial maxillectomy. • Post-operative radiotherapy is not necessary.

Question: 129
50 yrs old male presented with odynophagia, “hot potato” voice and
stridor.On examination diffuse swelling over the posterior pharyngeal
wall was seen. X-ray revealed increased retropharyngeal space. Most
suspected diagnosis is :

A Retropharyngeal abscess.

B Prevertebral abscess.

C Parapharyngeal abscess.

D Ludwig’s Angina

12.06% People got this right

Explanation:

Correct Answer (B) Prevertebral abscess. Ref: Read the text below Sol: •
Prevertebral abscess presents with diffuse swelling while retropharyngeal
abscess presents with localized swelling.

Question: 130
Vocal cord fixation is most likely due to invasion of the

https://emedicoz.com/testresult/683510s14692521 98/149
6/9/24, 12:55 PM Neet PG Preparation, Neet PG Coaching, FMGE, USMLE

A Conus elasticus.

B Quadrangular membrane.

C Paraglottic space.

D Broyle tendon.

16.96% People got this right

Explanation:

Correct Answer (C) Paraglottic space. Ref: Read the text below Sol: • There are
multiple barriers in the larynx to cancer spreading, including the conus elasticus,
quadrangular membrane, and the Broyle tendon. • Vocal cord fixation is usually
a sign of paraglottic space invasion or arytenoid involvement.

Question: 131
All of the following are contents of the posterior triangle of the neck
except:

A Spinal part of accessory


nerve

B Trunks of brachial plexus

C Internal jugular vein

D Transverse cervical artery

33.83% People got this right

Explanation:

https://emedicoz.com/testresult/683510s14692521 99/149
6/9/24, 12:55 PM Neet PG Preparation, Neet PG Coaching, FMGE, USMLE

Correct Answer (C)

Internal jugular vein

The posterior triangle is bounded posteriorly by the trapezius muscle, anteriorly


by the sternocleidomastoid muscle, and inferiorly by the clavicle.
The posterior cervical triangle is subdivided into the following triangles by the
inferior belly of the omohyoid muscle:

Occipital triangle, whose contents are:

Nerve to rhombideus

Cutaneous branches of cervical plexus of nerves

Spinal accessory nerve

Upper part of brachial plexus

Transverse cervical artery and vein

Cervical lymph nodes

Occipital artery

Supraclavicular nodes

Subclavian triangle (also known as Supraclavicular), whose contents are:

Third part of subclavian artery Part of subclavian vein Suprascapular artery and
vein Supraclavicular lymph nodes Three trunks of brachial plexus

Nerve to serratus anterior Nerve to subclavius Suprascapular nerve Lower part of


external jugular vein Transverse cervical artery and vein

Question: 132
A patient recovering from a traumatic injury to the face now finds himself
unable to chew his food. On physical examination, his jaw deviates to the
right side. The nerve that has been injured in this patient exits the skull
through which of the following foramina?

https://emedicoz.com/testresult/683510s14692521 100/149
6/9/24, 12:55 PM Neet PG Preparation, Neet PG Coaching, FMGE, USMLE

A Foramen rotundum

B Foramen ovale

C Foramen spinosum

D Foramen lacerum

41.61% People got this right

Explanation:

Correct Answer (B) Foramen ovale Ref: Read the text below Sol All the given
features indicate that Mandibular nerve is involved hence Foramen ovale is
correct

Question: 133
A 43-year-old male received a deep intramuscular injection two days ago
and now presents with difficulty walking. You note that his right hip
drops every time he raises his right foot. At which location did this
patient most likely receive his injection?

A Superomedial quadrant of
the buttock

B Superolateral quadrant of
the buttock

C Inferomedial quadrant of
the buttock

D Inferolateral quadrant of
the buttock

https://emedicoz.com/testresult/683510s14692521 101/149
6/9/24, 12:55 PM Neet PG Preparation, Neet PG Coaching, FMGE, USMLE

39.6% People got this right

Explanation:

Correct Answer (A)

Superomedial quadrant of the buttock

Exp:-

All injections into the gluteal region should target the superolateral quadrant, to avoid
damage to the gluteal nerves and sciatic nerve. This patient is exhibiting the classic
"gluteus medius gait," i.e. the hip dips downward when the ipsilateral foot is lifted off of
the ground. This gait is observed when there is injury to the superior gluteal nerve or to
the gluteus medius muscle itself. The superior gluteal nerve is derived from the L4-S1
ventral rami and leaves the pelvis through the greater sciatic foramen above the level of
the piriformis. Injection in the superomedial quadrant of the buttock is likely to injure
this nerve.

Question: 134
A 5-year-old male demonstrates persistent food-seeking behavior He is
obese, and his mother reports thathe sometimes exhibits aggressive and
bizarre behaviors These clinical findings may be explained by a lesionin
which of the following hypothalamic nuclei?

https://emedicoz.com/testresult/683510s14692521 102/149
6/9/24, 12:55 PM Neet PG Preparation, Neet PG Coaching, FMGE, USMLE

A Lateral

B Suprachiasmatic

C Supraoptic

D Ventromedial

30.8% People got this right

Explanation:

Correct Answer (D) Ventromedial Explanation: The body's satiety center resides
within the ventromedial nucleus of the hypothalamus. The neurons heredetect
blood glucose elevations and signal satiety in response. Bilateral lesions of the
ventromedial nucleicause hyperphagia and obesity. Episodes of rage and/or
savage behavior are also sometimes noted. Themost common cause of such
lesions is tumor invasion, for example by a craniopharyngioma. Lesions of the
ventromedial nuclei are the cause of obesity in only exceedingly rare cases.

Question: 135
A patient presents to your office with right leg numbness. When he walks,
you notice that he lifts his right foot higher than he does his left, and that
his right foot slaps to the ground with each step. On neurologic exam,
you also ascertain that he is unable to evert the right foot. Which of the
following nerves has most likely been injured?

A Tibial

B Common peroneal

C Superficial peroneal

https://emedicoz.com/testresult/683510s14692521 103/149
6/9/24, 12:55 PM Neet PG Preparation, Neet PG Coaching, FMGE, USMLE

D Sural

51.42% People got this right

Explanation:

Correct Answer (B) Common peroneal Exp: Patients with common peroneal
nerve damage present with an equinovarus (plantarflexed and inverted) posture
of the affected foot due to paralysis of the peroneus longus and peroneus brevis
muscles (mediate foot eversion), paralysis of the tibialis anterior muscle
(mediates dorsiflexion), and paralysis of the extrinsic extensors of the toes. Injury
to this nerve also causes loss of sensation to the anterolateral leg. The classic
finding on gait exam in patients with common peroneal nerve injury is "foot
drop," where the affected leg is lifted high off of the ground while walking due
to an inability to dorsiflex the foot. The affected foot will also classically slap to
the ground with each step.

Question: 136
An 15-year-old girl comes to the psychiatrist because she pulls out her
hair in pathes when she is anxious or upset. She is taught to make a tight
fist whenever she has this impulse rather than pull out her hair. Which of
the following techniques is this?

A Habit reversal training

B Extinction

C Flooding

D Desensitization

63.91% People got this right

Explanation:

https://emedicoz.com/testresult/683510s14692521 104/149
6/9/24, 12:55 PM Neet PG Preparation, Neet PG Coaching, FMGE, USMLE

Correct Answer (A) Habit reversal training It is a form of behavioural therapy to


reduce the impulse related behaviour, especially in trichotillomania.

Question: 137
30 year old gentleman has excessive sleep at work attributed to sleep
discomfort at night. He also has recent history of falling while partying
with friends. What are the other features that can be seen associated with
his condition?

A Restlessness of the legs


before going to bed

B Snoring with witnessed


sleep apnea

C Generalised seizure while


awake

D Paralysis with sleep awake


junction hallucinations

22.35% People got this right

Explanation:

Correct Answer (D) Paralysis with sleep awake junction hallucinations Excessive
sleep at work, sleep discomfort at night, history of falling while partying with
friends is suggestive of narcolepsy. In narcolepsy, we also get sleep paralysis,
and hypnagogic and hypnapompic hallucinations.

Question: 138
A 32-year-old woman presents to the psychiatrist. She states she is
depressed because she did this to her child. The infant has growth
retardation, microphthalmia, short palpebral fissures, midface hypoplasia,

https://emedicoz.com/testresult/683510s14692521 105/149
6/9/24, 12:55 PM Neet PG Preparation, Neet PG Coaching, FMGE, USMLE

a short philtrum, a thin upper lip, and microcephaly. Which is the most
likely diagnosis of the mother?

A Bipolar disorder

B Major depression

C Hypochondriasis

D Alcohol dependence

47.43% People got this right

Explanation:

Correct Answer (D) Alcohol dependence The features described in this clinical
scenario resembles foetal alcohol syndrome. • Fetal alcohol syndrome affects
about one third of all infants born to alcoholic women. • The syndrome is
characterized by growth retardation of prenatal origin (height, weight); minor
anomalies, including microphthalmia (small eyeballs), short palpebral fissures,
midface hypoplasia (under-development), a smooth or short philtrum, and a thin
upper lip; and central nervous system (CNS) manifestations, including
microcephaly (head circumference below the third percentile), a history of
delayed development, hyperactivity, attention deficits, learning disabilities,
intellectual deficits, and seizures. • The incidence of infants born with fetal
alcohol syndrome is about 0.5 per 1,000 live births. • Some studies suggest that
alcohol use during pregnancy may contribute to attention-deficit hyperactivity
disorder (ADHD).

Question: 139
A 25-month-old boy plays with a ball, which rolls under a couch. The boy
promptly crawls under the couch to retrieve the ball. According to
Piaget’s theories of cognitive development, which thinking process best
describes this child’s behaviour?

https://emedicoz.com/testresult/683510s14692521 106/149
6/9/24, 12:55 PM Neet PG Preparation, Neet PG Coaching, FMGE, USMLE

A Object permanence

B Initiative versus guilt

C Object constancy

D Sensorimotor stage

30.32% People got this right

Explanation:

Correct Answer (A) Object permanence By 2 years of age, child passes throught
sensorimotor stage and finally learns that objects are permanent. Remember :
Object constancy : Margaret Mahler Concept Object permanence : Jean Piaget
Concept

Question: 140
A 35-year-old woman sees a psychiatrist for a chief complaint of
depressed mood. She begins psychotherapy and sees the physician once
per week. After 3 months of therapy, she tells the psychiatrist that she is
very afraid of him because he is so angry and irritable person. She
behaves as if this is true and that the psychiatrist will explode with rage at
any minute. The psychiatrist is not normally seen as an angry person and
is unaware of any anger toward the patient. Which of the following
defense mechanisms is this patient likely displaying?

A Distortion

B Blocking

C Isolation

https://emedicoz.com/testresult/683510s14692521 107/149
6/9/24, 12:55 PM Neet PG Preparation, Neet PG Coaching, FMGE, USMLE

D Projection

42.82% People got this right

Explanation:

Correct Answer (D) Projection Projection, involves the tendency to see your own
unacceptable desires in other people.

Question: 141
A 78-year-old woman with a history of coronary artery disease & diabetes
takes aspirin 150 mg, telmisartan 40, glimepiride 2mg and clopidogrel 75
once a day from last 5 years. She is being taken for an emergency
exploratory laparotomy. Possible side effects associated with her drug
history include all the following EXCEPT

A Bradycardia

B Hypotension

C Hypoglycaemia

D Increased risk of bleeding

47.72% People got this right

Explanation:

Correct Answer (A)

https://emedicoz.com/testresult/683510s14692521 108/149
6/9/24, 12:55 PM Neet PG Preparation, Neet PG Coaching, FMGE, USMLE

DRUGS THAT NEED TO BE CONTINUED

• BETA BLOCKERS

• NTG

• CCBS

• STATINS

• STEROIDS

• THIAZIDES

• POP

• ANTI EPILEPTICS

• THYROID MEDS

• ATT

• ART

Question: 142
The following drugs are predominantly metabolized by the liver:

A Propofol

B Cisatracurium

https://emedicoz.com/testresult/683510s14692521 109/149
6/9/24, 12:55 PM Neet PG Preparation, Neet PG Coaching, FMGE, USMLE

C Esmolol

D Mivacurium

23.26% People got this right

Explanation:

Correct Answer (A) • Propofol, like other IV induction agents, is predominantly


metabolized via conjugation by the cytochrome P450 system in the liver. •
However, the rate of clearance of propofol exceeds the rate of hepatic blood
flow, suggesting that propofol undergoes some degree of extrahepatic
metabolism. • Cisatracurium, like its parent compound atracurium, does undergo
hepatic metabolism, however the majority of the drug undergoes Hofmann
elimination, a pH- and temperature dependent degradation of the drug. •
Esmolol has a very short duration of action. This is due to rapid metabolism via
ester hydrolysis by cholinesterase enzymes found in red blood cells. •
Mivacurium, like suxamethonium, is metabolized by butyrylcholinesterase (also
known as plasma cholinesterase and pseudocholinesterase, to differentiate it
from acetylcholinesterase). • Therefore, patients with a history of any form of
suxamethonium apnoea should not be given mivacurium.

Question: 143
Cisatracurium is preferred over atracurium because?

A Less potent and so less


laudonosine production

B No histamine release

C Metabolism by Hoffman’s
elimination

D Shorter duration of action.

26.38% People got this right


https://emedicoz.com/testresult/683510s14692521 110/149
6/9/24, 12:55 PM Neet PG Preparation, Neet PG Coaching, FMGE, USMLE

Explanation:

Correct Answer (B) • Atracurium is an amino steroid Non depolarising Muscle


Relaxant which a mixture of 10 different isomers. • Atracurium is metabolised by
2 organ independent processes of nonspecific esterase hydrolysis (66%) and
Hoffmann’s elimination (33%). • Hoffman’s elimination is a non-enzymatic organ
independent spontaneous degradation of the molecule under physiological ph.
and temperature. • Atracurium is associated with histamine release and therefore
causes transient hypotension, reflex tachycardia and bronchoconstriction. •
Cisatracurium is a pure cis isomer of atracurium and has some difference in
properties as compared to atracurium. 1. It is 4 times more potent than
atracurium 2. It is metabolized by only Hoffmann’s elimination 3. No histamine
release 4. No risk of laudonosine toxicity

Question: 144
Anesthetic gas with maximum respiratory irritation

A Halothane

B Enflurane

C Desflurane

D Sevoflurane

28.83% People got this right

Explanation:

Correct Answer (C) Inhaled anesthetic agent which in pungent in odour are: •
Ether • Isoflurane • Desflurane (maximum) Pungent inhaled anesthetic agents
cant be used for induction of anesthesia.

Question: 145

https://emedicoz.com/testresult/683510s14692521 111/149
6/9/24, 12:55 PM Neet PG Preparation, Neet PG Coaching, FMGE, USMLE

Regarding malignant hyperthermia, which of the following statements


are true?

A Has autosomal recessive


inheritance.

B Always occurs within the


first few hours of exposure to
triggering agent.

C Dantrolene 2.5 mg/kg IV


up to a maximum of 10 mg/kg is
the drug of choice.

D Masseter spasm is the first


sign.

49.4% People got this right

Explanation:

Correct Answer (C) •Malignant hyperthermia (MH) is a pharmacogenetic disease


of skeletal muscle induced by exposure to certain anaesthetic agents, such as
suxamethonium and all of the halogenated volatile anaesthetic agents. •It is
inherited as an autosomal dominant condition and caused by loss of normal Ca
2 + homeostasis at some point along the excitation–contraction coupling
process on exposure to triggering agents. •The most likely site is the triadic
junction between the T tubule, involving the voltage sensor of the
dihydropyridine receptor (DHPR), and the sarcoplasmic reticulum (SR), involving
the Ca 2 + efflux channel of the ryanodine receptor (RYR1). •Clinical diagnosis
can be difficult, as the presentation of MH varies. •Masseter spasm is a
premonitory sign of malignant hyperthermia. •Sudden and abrupt rise of ETCO2
is the 1st sign of MH. •Other signs are those of increased metabolism, like
tachycardia, dysrhythmias, increased CO 2 production, metabolic acidosis,
pyrexia, and DIC. •It can be a florid dramatic life-threatening event or have an
insidious onset. •It rarely develops 2–3 days postoperatively, with massive
myoglobinuria and/or renal failure due to severe rhabdomyolysis. •Treatment
consists of supportive management and IV Dantrolene 2.5 mg/kg bolus up to 10

https://emedicoz.com/testresult/683510s14692521 112/149
6/9/24, 12:55 PM Neet PG Preparation, Neet PG Coaching, FMGE, USMLE

mg/kg every 5 mins followed by an infusion of 1 mg/ kg every 6 hours for 24 to


48 hours.

Question: 146
A psychiatric person hears sounds of daggers being sharpened, while
normal people around him do not hear anything. This is an example of

A Illusion

B Obsessive compulsive
behavior

C Hallucination

D Fugue

68.28% People got this right

Explanation:

Correct Answer (C) Hallucination

Question: 147
The usual fatal dose of Opium is

A 2 micrograms

B 2 milligrams

C 20 milligrams

https://emedicoz.com/testresult/683510s14692521 113/149
6/9/24, 12:55 PM Neet PG Preparation, Neet PG Coaching, FMGE, USMLE

D 2 grams

24.89% People got this right

Explanation:

Correct Answer (D) 2 grams

Question: 148
Motorcyclist’s fracture is

A Ring fracture

B Division of base of skull


into two halves – an anterior and
a posterior

C Division of base of skull


into two halves – a left lateral and
a right lateral

D Comminuted fracture of
the vault

36.18% People got this right

Explanation:

Correct Answer (B) Division of base of skull into two halves – an anterior and a
posterior

Question: 149
Smokeless gunpowder is composed of

https://emedicoz.com/testresult/683510s14692521 114/149
6/9/24, 12:55 PM Neet PG Preparation, Neet PG Coaching, FMGE, USMLE

A KMnO4

B HCN

C Nitrocellulose

D Sulphur

42.05% People got this right

Explanation:

Correct Answer (C) Nitrocellulose

Question: 150
Caloric content of ethyl alcohol is

A 4 cal/g

B 4 cal/cc

C 7 cal/g

D 7 cal/cc

22.1% People got this right

Explanation:

Correct Answer (C) 7 cal/g

Question: 151

https://emedicoz.com/testresult/683510s14692521 115/149
6/9/24, 12:55 PM Neet PG Preparation, Neet PG Coaching, FMGE, USMLE

Arrange the following structures in skin in the order of top to bottom (from
most superficial to deepest)

1. Desmoglein

2. Hemidesmosomes

3. Collagen7

4. Laminin

A 1, 2, 3, 4

B 2, 1, 4, 3

C 2, 1, 3, 4

D 1, 2, 4, 3

28.93% People got this right

Explanation:

Correct Answer (D) 1, 2, 4, 3

Question: 152
A patient from Himachal Pradesh gets a thorn prick and subsequently
presents with a verrucous lesion. The diagnosis?

https://emedicoz.com/testresult/683510s14692521 116/149
6/9/24, 12:55 PM Neet PG Preparation, Neet PG Coaching, FMGE, USMLE

A Sporothrix

B Chromoblastomycosis

C Eumycetoma

D Verruca

43.15% People got this right

Explanation:

Correct Answer (B) Chromoblastomycosis

Question: 153
A child presents with the image shown. The diagnosis is?

https://emedicoz.com/testresult/683510s14692521 117/149
6/9/24, 12:55 PM Neet PG Preparation, Neet PG Coaching, FMGE, USMLE

A Bullous impetigo

B SSSS

C Candidal intertrigo

D Scrofuloderma

15.33% People got this right

Explanation:

Correct Answer (C)

Question: 154
Check the MDT schedule in adult leprosy and mark the correct options: A.
Rifampicin 600mg once a month supervised B. Clofazimine 300 mg once
a month supervised C. Dapsone 50 mg once a day unsupervised D.
Clofazimine 50 mg once a day unsupervised

A A, B, D are correct

B A and B are correct

C B and D are correct

D All four correct

28.93% People got this right

Explanation:
https://emedicoz.com/testresult/683510s14692521 118/149
6/9/24, 12:55 PM Neet PG Preparation, Neet PG Coaching, FMGE, USMLE

Correct Answer (A) A, B, D are correct

Question: 155
Arrange the following lesions in varicella in order of their time of onset in
the infection ( from earliest to last) 1. Crust 2. Vesicle 3. Pustule 4. Papule

A 1, 2, 3, 4

B 2, 1, 4, 3

C 2, 4, 3, 1

D 4, 2, 3, 1

42.19% People got this right

Explanation:

Correct Answer (D) 4, 2, 3, 1

Question: 156
Cytosine in DNA is deaminated, the uracil residue that results may be
removed by

A An endonuclease

B An exonuclease

C A glycosylase

D Polynucleotide ligase

https://emedicoz.com/testresult/683510s14692521 119/149
6/9/24, 12:55 PM Neet PG Preparation, Neet PG Coaching, FMGE, USMLE

26.09% People got this right

Explanation:

Correct Answer (A)

An endonuclease

Question: 157
Which one of the following sequences places the lipoproteins in the order
of most dense to least dense?

A
HDL/VLDL/chylomicrons/LDL

B
HDL/LDL/VLDL/chylomicrons

C
LDL/chylomicrons/HDL/VLDL

D
VLDL/chylomicrons/LDL/HDL

59.11% People got this right

Explanation:

Correct Answer (B)

HDL/LDL/VLDL/chylomicrons

Question: 158
Why glucose is stored as glycogen

https://emedicoz.com/testresult/683510s14692521 120/149
6/9/24, 12:55 PM Neet PG Preparation, Neet PG Coaching, FMGE, USMLE

A Has compact structure

B Can supply glucose for one


week of fasting

C Has many reducing ends

D Can be stored in multiple


organs

34.45% People got this right

Explanation:

Correct Answer (A)

Has compact structure

Question: 159
Enzyme which is active in well fed state is:

A Carnitine palmitoy1
transferase 1

B Pyruvate carboxylase

C Acety1 CoA carboxylase

D HMG lyase

27.1% People got this right

Explanation:

Correct Answer (C)


https://emedicoz.com/testresult/683510s14692521 121/149
6/9/24, 12:55 PM Neet PG Preparation, Neet PG Coaching, FMGE, USMLE

Acety1 CoA carboxylase

Question: 160
A 10 year old boy presented with abdominal pain, muscle weakness and
fatigue. On investigations , serum lead levels were found increased in
blood . Activity of which of the following enzyme in the liver is increased :

A ALA synthase

B Ferrochelatase

C PGB deaminase

D Heme oxygenase

36.81% People got this right

Explanation:

Correct Answer (A)

ALA synthase

Question: 161
A 40-year-old male meets with a RTA after rash driving his bike without the
helmet on. NCCT head shows.

https://emedicoz.com/testresult/683510s14692521 122/149
6/9/24, 12:55 PM Neet PG Preparation, Neet PG Coaching, FMGE, USMLE

A SDH

B SAH

C EDH

D Cerebral contusion

16.15% People got this right

Explanation:

Correct Answer (C)

EDH

The NCCT head shows a biconvex hyper-density in right frontal area suggestive
of extra-dural hemorrhage. Also visible is subperiosteal hemorrhage on right
side.

Question: 162
Identify the X-ray view shown.

https://emedicoz.com/testresult/683510s14692521 123/149
6/9/24, 12:55 PM Neet PG Preparation, Neet PG Coaching, FMGE, USMLE

A Caldwell

B Water's

C Schuller

D Towne

2.98% People got this right

Explanation:

Correct Answer (A)

Caldwell

Notice that only the frontal sinus and anterior ethmoidal sinuses are visible and
this is seen in Caldwell view.

https://emedicoz.com/testresult/683510s14692521 124/149
6/9/24, 12:55 PM Neet PG Preparation, Neet PG Coaching, FMGE, USMLE

Question: 163
A 35-year-old female presents with dysphagia, regurgitation and weight
loss. What is the likely diagnosis?

A Diffuse esophageal spasm

B Achalasia cardia

C Malignancy

D Leiomyoma

14.37% People got this right

Explanation:

Correct Answer (B) Achalasia cardia • Notice the abrupt tapering at the distal
end of esophagus. The esophagus also looks grossly dilated just proximal to
tapering. This is seen in Diffuse esophageal spasm. • Option A would have
corkscrew appearance • Option C will have irregular filling defect in lower or
middle part of esophagus • Option D will produce a local filling defect with
relatively smooth margins in any part of esophagus

Question: 164

https://emedicoz.com/testresult/683510s14692521 125/149
6/9/24, 12:55 PM Neet PG Preparation, Neet PG Coaching, FMGE, USMLE

Identify the given IVP image:

A Ureterocele

B Hydronephrosis

C Staghorn calculi

D Bladder calculi

15.04% People got this right

Explanation:

Correct Answer (A) Ureterocele The given IVP shows bilateral cobra head
appearance of distal ureter diagnostic of ureterocele.

Question: 165
Gold standard investigation for sinuses is:

A X-ray

B MRI

https://emedicoz.com/testresult/683510s14692521 126/149
6/9/24, 12:55 PM Neet PG Preparation, Neet PG Coaching, FMGE, USMLE

C CT scan

D PET scan

29.26% People got this right

Explanation:

Correct Answer (C) CT scan • Functional endoscopic sinus surgery is a minimally


invasive technique used to restore sinus ventilation and normal function. •
Fiberoptic telescopes are used for diagnosis and during the procedure, and
computed tomography is used to assess the anatomy and identify diseased
areas. • The most suitable candidates for this procedure have recurrent acute or
chronic infective sinusitis, and an improvement in symptoms of up to 90 percent
may be expected following the procedure.

Question: 166
Early onset IBD which is refractory to treatment is due to deficiency of
which cytokine

A IL-1

B IL-6

C IL-10

D IL-12

13.46% People got this right

Explanation:

Correct Answer (C) IL-10

https://emedicoz.com/testresult/683510s14692521 127/149
6/9/24, 12:55 PM Neet PG Preparation, Neet PG Coaching, FMGE, USMLE

Question: 167
For diagnosis of IBS abdomen pain frequency of >1day/week should be
present for

A >1month

B >2 months

C >3 months

D >6 months

44.98% People got this right

Explanation:

Correct Answer (C)

>3 months

Question: 168
A patient has Hepatitis B surface antigen (HBsAg) positive, Antibody to
HBsAg (anti-HBs) negative Antibody to hepatitis B core antigen (anti-
HBc) IgM negative, Anti-HBc IgG positive What is the next step in his
management?

A Check HBV DNA

B Liver biopsy

C Repeat blood tests in 6


months

https://emedicoz.com/testresult/683510s14692521 128/149
6/9/24, 12:55 PM Neet PG Preparation, Neet PG Coaching, FMGE, USMLE

D Tenofovir

39.36% People got this right

Explanation:

Correct Answer (A) Check HBV DNA

Question: 169
A 45 year old woman presents with a 5-day history of pale stools and
dark urine associated with cramping epigastric and right upper quadrant
pains. Blood tests show elevated ALP and GGT. What would be your first-
line imaging investigation?

A CT pancreas

B MRCP

C ERCP

D Ultrasound abdomen

49.98% People got this right

Explanation:

Correct Answer (D) Ultrasound abdomen

Question: 170
A 14 year old boy has behaviour abnormalities with tremors.On examination
brownish green coloured deposit at corneal periphery is observed as shown
in the figure.What is the likely etiology

https://emedicoz.com/testresult/683510s14692521 129/149
6/9/24, 12:55 PM Neet PG Preparation, Neet PG Coaching, FMGE, USMLE

A Hemochromatosis

B Wilson disease

C Sydenham’s chorea

D Acute viral hepatitis

58.87% People got this right

Explanation:

Correct Answer (B) Wilson disease

Question: 171
A 33-year-old man comes for donating blood. He is asymptomatic. On
further history he has never been vaccinated for hepatitis B. Serologic
tests reveal negative hepatitis B surface antigen (HBsAg) and positive
Anti HBs and IgG antiHBc. Which of the following conditions does this
serologic pattern represents?

A Previous hepatitis B
infection

https://emedicoz.com/testresult/683510s14692521 130/149
6/9/24, 12:55 PM Neet PG Preparation, Neet PG Coaching, FMGE, USMLE

B Chronic hepatitis

C Acute hepatitis

D Precore mutant

52.62% People got this right

Explanation:

Correct Answer (A) Previous hepatitis B infection

Question: 172
A 20-year-old woman with past history of rheumatic fever has dyspnea.
On examination- loud S1 present, opening snap and mid diastolic
rumbling murmur heard at the apex. What is the cause of this
presentation?

A Mitral stenosis

B Mitral regurgitation

C Aortic stenosis

D Aortic regurgitation

55.6% People got this right

Explanation:

Correct Answer (A) Mitral stenosis

Question: 173
https://emedicoz.com/testresult/683510s14692521 131/149
6/9/24, 12:55 PM Neet PG Preparation, Neet PG Coaching, FMGE, USMLE

A 55-year-old female developed chest pain with ST elevation in ECG.


Coronary angiography revealed no thrombus. On echocardiography there
is apex dilatation in systole. What is the likely diagnosis

A HOCM

B Restrictive cardiomyopathy

C ST elevation Myocardial
infarction

D Tako Tsubo
cardiomyopathy

42.96% People got this right

Explanation:

Correct Answer (D) Tako Tsubo cardiomyopathy

Question: 174
A 76-year-old woman has atrial fibrillation with history of hypertension
and diabetes. There is no valvular heart disease. Which of the following
would be the most appropriate antithrombotic regimen for this patient?

A Aspirin

B Aspirin plus clopidogrel

C Apixaban

D Anti thrombotic not


required in the patient

https://emedicoz.com/testresult/683510s14692521 132/149
6/9/24, 12:55 PM Neet PG Preparation, Neet PG Coaching, FMGE, USMLE

13.41% People got this right

Explanation:

Correct Answer (C) Apixaban

Question: 175
A 40 year old patient has developed palpitations.Pulse:130 beats/minute,
irregularly irregular with positive pulse deficit. What is the likely cause of
patient’s tachycardia?

A Sinus tachycardia

B Atrial fibrillation

C PSVT

D Atrial flutter

49.98% People got this right

Explanation:

Correct Answer (B) Atrial fibrillation

Question: 176
A 40-year old woman presents with sudden onset palpitations. An ECG
done reveals narrow QRS complex tachycardia. The most likely diagnosis
is:

A Ventricular tachycardia

B Pre-excited tachycardia

https://emedicoz.com/testresult/683510s14692521 133/149
6/9/24, 12:55 PM Neet PG Preparation, Neet PG Coaching, FMGE, USMLE

C Paroxysmal
supraventricular tachycardia with
aberrancy

D AV rentry tachycardia

10.28% People got this right

Explanation:

Correct Answer (D) AV rentry tachycardia

Question: 177
A 36-year-old man has generalized swelling over the body. Urinalysis: 4+
protein, oval fat bodies, serum albumin 2.1 g/dL, Liver function tests:
normal, serum Low-density lipoprotein (LDL) 145 mg/dL; triglycerides 500
mg/dL (elevated).What is the likely cause of this patient presentation

A Chronic liver disease

B Hypothyroidism

C CHF

D Nephrotic syndrome

64.2% People got this right

Explanation:

Correct Answer (D) Nephrotic syndrome

Question: 178

https://emedicoz.com/testresult/683510s14692521 134/149
6/9/24, 12:55 PM Neet PG Preparation, Neet PG Coaching, FMGE, USMLE

A patient is diagnosed with membranous glomerulopathy and is started


on prednisone orally but after 3 months there is no improvement in urine
protein excretion or serum albumin levels. What should you add to
therapy?

A Add cyclophosphamide.

B Add cyclosporine.

C Add rituximab.

D Add tacrolimus.

37.43% People got this right

Explanation:

Correct Answer (A) Add cyclophosphamide.

Question: 179
A male patient history of hypertension ,diabetes and CKD. He is taking
Lisinopril, amlodipine, Linagliptin and Gliclazide. On examination- BP:
150/96 mm Hg. Lab tests- serum creatinine 4 mg/dL; potassium 4.9
mEq/L;HbA1c 8 %, urine dipstix - protein 2+,What will you do next

A Observation with serial


kidney function tests

B Increase anti Diabetes and


antihypertensive drugs

C Immediate dialysis

https://emedicoz.com/testresult/683510s14692521 135/149
6/9/24, 12:55 PM Neet PG Preparation, Neet PG Coaching, FMGE, USMLE

D Renal transplant

19.69% People got this right

Explanation:

Correct Answer (B) Increase anti Diabetes and antihypertensive drugs

Question: 180
A 24 year female comes for a routine check up.Her thyroid function test is
as follows. Serum T3 = Normal, T4 = Normal, TSH = 10mU/L. What is the
most likely diagnosis?

A Primary hypothyroidism

B Secondary hypothyroidism

C Subclinical hypothyroidism

D Non thyroidal illness

54.77% People got this right

Explanation:

Correct Answer (C) Subclinical hypothyroidism

Question: 181
A 68 year old man diagnosed as having type 2 diabetes, hypertension,
dyslipidemia, CHF and microalbuminuria. The patient is given
prescriptions for lisinopril, metformin,thiazide and a statin.The patient’s
glucose levels are not fully controlled.He is not willing for any injectable
therapy. What will be next appropriate agent

https://emedicoz.com/testresult/683510s14692521 136/149
6/9/24, 12:55 PM Neet PG Preparation, Neet PG Coaching, FMGE, USMLE

A Sulfonylurea

B Pioglitazone

C GLP-1 agonist

D SGLT inhibitors

29.76% People got this right

Explanation:

Correct Answer (D) SGLT inhibitors

Question: 182
A 48-year-old man noticed that his shoe and ring size started increasing
over the last 1 to 2 years. On examination BP: 160/100 mm Hg; General
appearance: large nose, lips, and jaw; teeth widely spaced. Thyromegaly
present. Chest-clear, CVS- normal. What is the best initial test?

A Insulin like growth factor


(IGF)

B GH

C MRI head

D Glucose suppression test

43.12% People got this right

Explanation:

https://emedicoz.com/testresult/683510s14692521 137/149
6/9/24, 12:55 PM Neet PG Preparation, Neet PG Coaching, FMGE, USMLE

Correct Answer (A) Insulin like growth factor (IGF)

Question: 183
A 22-year old male is admitted with fever and altered consciousness for
two days. Examination reveals nuchal rigidity and diffuse petechial rash
on the trunk and lower extremities. The most likely etiologic agent is:

A Escherichia coli

B Streptococcus pneumoniae

C Neisseria meningitidis

D Haemophilus influenzae

59.27% People got this right

Explanation:

Correct Answer (C) Neisseria meningitidis

Question: 184
A 44-year-old man presents with involuntary movements of his face,
shoulders, and arms. His father had a similar condition. There is also a
history of mood changes for the past 6 months. On examination, he
appears restless with intermittent slow movements of his hands and face.
He has difficulty performing rapid finger movements, and tone is
decreased in the upper and lower limbs. Which of the following is the
most likely diagnosis?

A Parkinson’s disease

https://emedicoz.com/testresult/683510s14692521 138/149
6/9/24, 12:55 PM Neet PG Preparation, Neet PG Coaching, FMGE, USMLE

B Huntington’s chorea

C Amyotrophic lateral
sclerosis

D Sydenham’s chorea

42.55% People got this right

Explanation:

Correct Answer (B) Huntington’s chorea

Question: 185
A 45-year-old man presents with weakness and fasciculations in his arms
and legs. His cranial nerves are normal, but there is weakness of his left
handgrip and right leg quadriceps with loss of muscle bulk. There are
obvious fasciculations over the left forearm and right thigh. Tone is
increased in the arms and legs and the reflexes are brisk. There is no
sensory loss. Which of the following is the most likely diagnosis?

A Amyotrophic lateral
sclerosis

B Myotonic muscular
dystrophy

C Tabes dorsalis

D Migraine

35.46% People got this right

Explanation:

https://emedicoz.com/testresult/683510s14692521 139/149
6/9/24, 12:55 PM Neet PG Preparation, Neet PG Coaching, FMGE, USMLE

Correct Answer (A) Amyotrophic lateral sclerosis

Question: 186
Which one of the following is the minimum spirometry criterion for
diagnosis of bronchial asthma?

A Decrease in FEV1 ≥ 20 %
after 6 minutes of exercise

B Increase in FEV1 ≥ 12 %
following administration of
inhaled bronchodilators

C Increase in 500 ml of FEV1


after glucocorticoids inhalation

D Less than 20 % diurnal


variation on more than 3 days a
week on PEF diary

58.36% People got this right

Explanation:

Correct Answer (B) Increase in FEV1 ≥ 12 % following administration of inhaled


bronchodilators

Question: 187
A 50-year-old woman presented with pain in her hands for the past year.
The pain is worse in the morning, but improves as she starts to move
around over the course of the day over the next 1 to 2 hours. Reports:
ESR: 68 mm/h, CRP: elevated, Anti-CCP: positive. What is the most likely
diagnosis

https://emedicoz.com/testresult/683510s14692521 140/149
6/9/24, 12:55 PM Neet PG Preparation, Neet PG Coaching, FMGE, USMLE

A Rheumatoid arthritis

B Pulmonary TB

C SLE

D Sarcoidosis

67.47% People got this right

Explanation:

Correct Answer (A) Rheumatoid arthritis

Question: 188
A 10-year-old child has recurrent symptoms of joint pains and colicky
abdomen pain. Now he developed swelling of face and low urine output
as noticed by his parents. On examination BP-150/90 mmHg, palpable
purpura on the buttocks are present. Lab tests elevated BUN, serum
creatinine and thrombocytosis is observed. Urine tests revealed
microscopic hematuria. Which of the following is the most likely
diagnosis?

A Wegener’s granulomatosis

B Henoch Schönlein purpura

C Polyarteritis Nodosa

D Juvenile RA

61.62% People got this right

Explanation:
https://emedicoz.com/testresult/683510s14692521 141/149
6/9/24, 12:55 PM Neet PG Preparation, Neet PG Coaching, FMGE, USMLE

Correct Answer (B) Henoch Schönlein purpura

Question: 189
A 40 year old male patient has macrocytosis on complete blood count. He
can have all the following conditions EXCEPT:

A Liver disease

B Post splenectomy

C Hypothyroidism

D Myelodysplastic syndrome

16.58% People got this right

Explanation:

Correct Answer (B) Post splenectomy

Question: 190
In HIV infection IRIS (Immune reconstitution inflammatory syndrome)
usually develops when ART is started at what CD4 count

A Less than 50

B Less than 500

C More than 500

D More than 750

https://emedicoz.com/testresult/683510s14692521 142/149
6/9/24, 12:55 PM Neet PG Preparation, Neet PG Coaching, FMGE, USMLE

25.35% People got this right

Explanation:

Correct Answer (A) Less than 50

Question: 191
In a study dye A is used to measure cardiac output and plasma volume.
The dye is replaced with a new dye which diffuses more rapidly out of the
capillaries. This would affect-

A Normal cardiac output,


altered plasma volume

B Altered cardiac output,


altered plasma volume

C Normal cardiac output,


false high plasma volume

D Normal cardiac output,


false low plasma volume

22.28% People got this right

Explanation:

Correct Answer (C) Cardiac output will be normal as the dye moves into the
interstitial space but as the dye moves out of the circulation concentration of the
dye will reduce in circulation so false high plasma volume will be estimated ie.
A= C X V according to principle of volume distribution of body fluids. So, V= A/C
as C decreases A will be falsely high.

Question: 192
True regarding excitation contraction coupling in smooth muscle is –

https://emedicoz.com/testresult/683510s14692521 143/149
6/9/24, 12:55 PM Neet PG Preparation, Neet PG Coaching, FMGE, USMLE

A Presence of troponin is
essential

B Sustained contraction can


occur with high Ca2+
concentration

C Phosphorylation of actin is
required for contraction

D Presence of cellular Ca2+ is


essential to cause muscle
contraction

27.5% People got this right

Explanation:

Correct Answer (D) Explanation- Ca2+ is essential for muscle contraction, and
troponin is absent in smooth muscle also phosphorylation of myosin not actin is
essential for muscle contraction.

Question: 193
A substance is prepared that specifically and completely inhibits glucose
transport in PCT. In healthy volunteer receiving this substance, clearance
of glucose will approximate the clearance of which of the following?

A PAH

B Urea

C Na+

D Inulin

https://emedicoz.com/testresult/683510s14692521 144/149
6/9/24, 12:55 PM Neet PG Preparation, Neet PG Coaching, FMGE, USMLE

42.36% People got this right

Explanation:

Correct Answer (D) Explanation- Glucose is filtered and completely reabsorbed


in PCT, if glucose reabsorption is inhibited then it will be completely filtered just
like inulin.

Question: 194
All of these are actions of atrial natriuretic peptide except:

A Afferent arteriolar
dilatation

B Mesangial cell constriction

C Decreased sodium
absorption in the PCT

D Inhibition of sodium
reabsorption in medullary
collecting duct

21.13% People got this right

Explanation:

Correct Answer (B) Explanation- ANP causes mesangial relaxation and dilatation
of afferent arterioles leading to increased glomerular filtration and sodium
excretion. It also acts on renal tubules to inhibit sodium reabsorption.

Question: 195
Following a sympathectomy, a 66-yearold man experiences orthostatic
hypotension. The explanation for this occurrence is-

https://emedicoz.com/testresult/683510s14692521 145/149
6/9/24, 12:55 PM Neet PG Preparation, Neet PG Coaching, FMGE, USMLE

A An exaggerated response
of the renin–angiotensin–
aldosterone system

B A suppressed response of
the renin–angiotensin–
aldosterone system

C An exaggerated response
of the baroreceptor mechanism

D A suppressed response of
the baroreceptor mechanism

27.84% People got this right

Explanation:

Correct Answer (D) Explanation- with sympathectomy efferent nerve component


of baroreceptor will not work so cannot correct fall in hypotension.

Question: 196
This classical X-ray sign is seen in which of the following conditions :

A Anterior Shoulder
Dislocation

https://emedicoz.com/testresult/683510s14692521 146/149
6/9/24, 12:55 PM Neet PG Preparation, Neet PG Coaching, FMGE, USMLE

B Posterior Shoulder
Dislocation

C Inferior Shoulder
Dislocation

D Central Shoulder
Dislocation

5.37% People got this right

Explanation:

Correct Answer (C)

Inferior Shoulder Dislocation

Question: 197
The most common cause of relapse after successful correction of CTEV is :

A Overcorrection during
Ponseti’s technique

B Inadequate number of POP


cast application

C Poor compliance to shoes


and splint by parents

D All of the above

32.97% People got this right

Explanation:

https://emedicoz.com/testresult/683510s14692521 147/149
6/9/24, 12:55 PM Neet PG Preparation, Neet PG Coaching, FMGE, USMLE

Correct Answer (C) Poor compliance to shoes and splint by parents

Question: 198
Most common complication of fracture neck of talus :

A AVN of head of talus

B AVN of neck of talus

C Subtalar arthritis

D Non union

19.5% People got this right

Explanation:

Correct Answer (C) Subtalar arthritis

Question: 199
A 12 year old boy has been complaining of pain and swelling in his right
thigh. He has associated fever as well. He was taken to a doctor where
blood markers were done which show raised TLC , ESR and CRP. He is put
on IV antibiotics and NSAID. He responds to the medicines till the time he
takes them but then symptoms occur again when he stops. X-ray was
done which shows laminated periosteal reaction along the diaphysis of
femur. Which of the following will be the most conclusive investigation to
establish the diagnosis ?

A Bone Scan

B MRI

https://emedicoz.com/testresult/683510s14692521 148/149
6/9/24, 12:55 PM Neet PG Preparation, Neet PG Coaching, FMGE, USMLE

C Biopsy

D CT scan with 3D
reconstruction

35.17% People got this right

Explanation:

Correct Answer (C) Biopsy

Question: 200
Most common joint involved in Septic Arthritis :

A Knee

B Hip

C Wrist

D Elbow

44.99% People got this right

Explanation:

Correct Answer (A) Knee

https://emedicoz.com/testresult/683510s14692521 149/149

You might also like